Executive Assitant : Date - 21/11/2022 Shift - 2 Previous Year Paper Full Length PDF


Question 1


Which of the following statements is FALSE with reference to the Visual Display Unit (VDU) of a computer system?

कंप्यूटर सिस्टम की विजुअल डिस्प्ले यूनिट (VDU) के संदर्भ में निम्नलिखित में से कौन सा कथन गलत है?

Options

A

The shape of pixels determines the image’s sharpness displayed on the VDU.

The shape of pixels determines the image’s sharpness displayed on the VDU.

B

Cathode-Ray Tube and Flat-Panel Display are variations in VDUs

Cathode-Ray Tube and Flat-Panel Display are variations in VDUs

C

VDU creates images by arranging small dots, known as pixels, in a rectangular pattern.

VDU creates images by arranging small dots, known as pixels, in a rectangular pattern.

D

VDU is a computer’s primary output device.

VDU is a computer’s primary output device.


Solution:

Correct Answer:

A

The shape of pixels determines the image’s sharpness displayed on the VDU.

पिक्सेल का आकार वीडीयू (VDU) पर डिस्प्ले हो रही इमेज की स्पष्टता को निर्धारित करता है।


Question 2


What keyboard shortcut is used to open the font dialog box in MS-PowerPoint 365?

एमएस-पॉवरपॉइंट 365 (MS-PowerPoint 365) में फॉन्ट डायलॉग बॉक्स को खोलने के लिए किस कीबोर्ड शॉर्टकट का उपयोग किया जाता है?

Options

A

Ctrl + N

Ctrl + N

B

Ctrl + T

Ctrl + T

C

Ctrl + M

Ctrl + M

D

Ctrl + F

Ctrl + F


Solution:

Correct Answer:

B

Ctrl + T

Ctrl + T


Question 3


With reference to the internet what is the full form of ARPANET?

इंटरनेट के संदर्भ में, ARPANET का पूर्ण रूप क्या है?

Options

A

Advanced Research Projects Agency Network 

Advanced Research Projects Agency Network 

B

Advanced Research Planning Agency Network

Advanced Research Planning Agency Network

C

Advanced Remedial Projects Agency Network 

Advanced Remedial Projects Agency Network 

D

Automatic Research Projects Agency Network

Automatic Research Projects Agency Network


Solution:

Correct Answer:

A

Advanced Research Projects Agency Network 

Advanced Research Projects Agency Network (एडवांस्ड रसर्च प्रोजेक्ट्स एजेंसी नेटवर्क)


Question 4


Which of the following groups of the Home tab contains the Bullets and Numbering feature in MS-Word 365?

एमएस-वर्ड 365 (MS-Word 365) में होम टैब के निम्न में से किस समूह में बुलेट्स एंड नंबरिंग (Bullets and Numbering) फीचर शामिल होता है?

Options

A

Paragraph

Paragraph

B

Styles

Styles

C

Clipboard

Clipboard

D

Editor

Editor


Solution:

Correct Answer:

A

Paragraph

पैराग्राफ (Paragraph)


Question 5


Which of the following options from Print Layout settings can be used to print four slides in one page in MS-PowerPoint 365?

एमएस-पॉवरपॉइंट 365 (MS-PowerPoint 365) में एक पेज में चार स्लाइड्स को प्रिंट करने के लिए प्रिंट लेआउट सेटिंग्स से निम्नलिखित में से कौन सा विकल्प इस्तेमाल किया जा सकता है?

Options

A

Outline

Outline

B

Full Page Slides

Full Page Slides

C

Handouts

Handouts

D

Notes Pages

Notes Pages


Solution:

Correct Answer:

C

Handouts

हैंडआउट्स (Handouts)


Question 6


Representation data in ASCII format in a computer system requires how many bits for each character?

कंप्यूटर सिस्टम में एएससीआईआई (ASCII) फॉर्मेट में रिप्रेजेंटेशन डेटा के लिए प्रत्येक कैरेक्टर के लिए कितने बिट की आवश्यकता होती है?

Options

A

7

7

B

10

10

C

9

9

D

5

5


Solution:

Correct Answer:

A

7

7


Question 7


Which of the following charts is ideal for showing trends in data at equal intervals, like months, quarters, or fiscal years in MS-Excel 365?

एमएस-एक्सेल 365 (MS-Excel 365) में बराबर अंतरालों, जैसे महीनों, तिमाहियों या वित्तीय वर्षों में डेटा में ट्रेंड्स दिखाने के लिए निम्न में से कौन सा चार्ट उपयुक्त होता है?

Options

A

Column

Column

B

Pie

Pie

C

Line

Line

D

Map

Map


Solution:

Correct Answer:

C

Line

लाइन (Line)


Question 8


Which of the following keyboard shortcut is used to quickly access the bookmarks creation menu in all major web browsers?

निम्नलिखित में से कौन सा कीबोर्ड शॉर्टकट, सभी प्रमुख वेब ब्राउज़र में बुकमार्क क्रिएशन मेनू को तुरंत एक्सेस करने के लिए उपयोग किया जाता है?

Options

A

Esc + D

Esc + D

B

Alt + D

Alt + D

C

Ctrl + D

Ctrl + D

D

Shift + D

Shift + D


Solution:

Correct Answer:

C

Ctrl + D

Ctrl + D


Question 9


What is the full form of IECT?

आईईसीटी (IECT) का पूर्ण रूप क्या है?

Options

A

Information Electronics and Communication Transmission

Information Electronics and Communication Transmission

B

Information Electronics and Communication Technology

Information Electronics and Communication Technology

C

Information Electronics and Conversation Technology

Information Electronics and Conversation Technology

D

Information Electric and Correspondence Transmission

Information Electric and Correspondence Transmission


Solution:

Correct Answer:

B

Information Electronics and Communication Technology

Information Electronics and Communication Technology (इन्फॉर्मेशन इलेक्ट्रॉनिक्स एंड कम्‍युनिकेशन टेक्नोलॉजी)


Question 10


Which of the following tabs of the Mouse component in the Control Panel should be used to select pointer speed of a mouse in Windows 10?

कंट्रोल पैनल में माउस कंपोनेंट का निम्न में से कौन सा टैब विंडोज़ 10 में माउस की पॉइंटर स्पीड को सेलेक्ट करने के लिए इस्तेमाल किया जाना चाहिए?

Options

A

Buttons

Buttons

B

Pointer options

Pointer options

C

Hardware

Hardware

D

Pointers

Pointers


Solution:

Correct Answer:

B

Pointer options

पॉइंटर ऑप्शन्स (Pointer options)


Question 11


Which of the following is true with reference to a Wide Area Network (WAN)?

वाइड एरिया नेटवर्क (WAN) के संदर्भ में निम्नलिखित में से कौन सा कथन सत्य है?

Options

A

There is only one type of WAN known as switched WAN.

There is only one type of WAN known as switched WAN.

B

The fault tolerance of a WAN is very high.

The fault tolerance of a WAN is very high.

C

WAN’s data rate is higher than LAN’s speed.

WAN’s data rate is higher than LAN’s speed.

D

Due to long distance transmission, the noise and error tend to be more in WAN.

Due to long distance transmission, the noise and error tend to be more in WAN.


Solution:

Correct Answer:

D

Due to long distance transmission, the noise and error tend to be more in WAN.

लंबी दूरी के ट्रांसमिशन के कारण, WAN में नॉइज़ (रव) और एरर (त्रुटि) अधिक होता है।


Question 12


Which of the following operators can be used to search for the exact phrase in the Google search engine?

निम्नलिखित में से किस ऑपरेटर का उपयोग गूगल सर्च इंजन में सटीक वाक्यांश खोजने के लिए किया जा सकता है?

Options

A

Quotations

Quotations

B

period

period

C

Tilde

Tilde

D

Minus

Minus


Solution:

Correct Answer:

A

Quotations

कोटेशन्स (Quotations)


Question 13


Which of the following features of the Data tab in MS-Excel 365 allows you to set up certain rules that dictate what can be entered into a cell?

एमएस-एक्सेल 365 (MS-Excel 365) में डेटा टैब की निम्न में से कौन-सा फीचर आपको कुछ नियम सेट करने की सुविधा देता है जो यह निर्धारित करते हैं कि सेल में क्या दर्ज किया जा सकता है?

Options

A

Data validation

Data validation

B

Data sorting

Data sorting

C

Data filtering

Data filtering

D

Data merging

Data merging


Solution:

Correct Answer:

A

Data validation

डेटा वैलिडेशन (Data validation)


Question 14


Which of the following types of files are NOT allowed to be sent as an attachment by most of the e-mail service providers?

अधिकांश ई-मेल सेवा प्रदाताओं द्वारा निम्नलिखित में से किस प्रकार की फाइलों को अटैचमेंट के रूप में भेजने की अनुमति नहीं होती है?

Options

A

.doc

.doc

B

.pdf

.pdf

C

.exe

.exe

D

.jpg

.jpg


Solution:

Correct Answer:

C

.exe

.ईएक्सई (.exe)


Question 15


‘Computer is a dumb machine and it cannot do any work without instruction from the user’— this statement refers to which of the following computer characteristics?

कंप्यूटर एक डंब मशीन है और यह यूज़र के निर्देश के बिना कोई काम नहीं कर सकता'- यह कथन निम्नलिखित में से किस कंप्यूटर विशेषता को दर्शाता है?

Options

A

Versatility

Versatility

B

No IQ

No IQ

C

No feeling

No feeling

D

Diligence

Diligence


Solution:

Correct Answer:

B

No IQ

कोई आईक्‍यू नहीं (No IQ)


Question 16


Which of the following is an area of mailbox in e-mail where the messages which are in the process of sending or which are failed to send are stored in general?

निम्न में से कौन सा ई-मेल में मेलबॉक्स का एक वह क्षेत्र है जहां वे संदेश, जो भेजने की प्रक्रिया में होते हैं या जो भेजने में विफल हो जाते हैं, सामान्य रूप से स्टोर किए जाते हैं?

Options

A

Sent mail

Sent mail

B

Outbox

Outbox

C

Inbox

Inbox

D

Spam

Spam


Solution:

Correct Answer:

B

Outbox

आउटबॉक्स (Outbox)


Question 17


Which of following options of the Transitions tab is used to change how the transition occurs for e.g., the direction from which the slide enters in MS-PowerPoint 365?

ट्रांज़िशन टैब का निम्न में से कौन सा विकल्प, ट्रांज़िशन कैसे होता है यह बदलने के लिए उपयोग किया जाता है, उदाहरण के लिए, वह दिशा जिससे स्लाइड एमएस-पॉवर पॉइंट 365 (MS-PowerPoint 365) में एंटर करती है?

Options

A

Animation

Animation

B

Duration

Duration

C

Effect Options

Effect Options

D

Advance Slide

Advance Slide


Solution:

Correct Answer:

C

Effect Options

इफेक्ट ऑप्शन्स (Effect Options)


Question 18


Which of the following is the file extension for a PC batch file in the Windows operating system?

विंडोज ऑपरेटिंग सिस्टम में पीसी (PC) बैच फाइल के लिए फाइल एक्सटेंशन निम्न में से कौन सा है?

Options

A

.BMP

.BMP

B

.BAT

.BAT

C

.CSV

.CSV

D

.DBF

.DBF


Solution:

Correct Answer:

B

.BAT

.BAT


Question 19


Which of the following is the correct order to insert columns in MS-Excel 365?

एमएस-एक्सेल 365 (MS-Excel 365) में कॉलम इन्सर्ट करने का सही क्रम निम्न में से कौन सा है?

Options

A

Home > Insert > Insert Sheet Columns

Home > Insert > Insert Sheet Columns

B

Insert > Insert Sheet Columns

Insert > Insert Sheet Columns

C

Draw > Insert > Insert Sheet Columns

Draw > Insert > Insert Sheet Columns

D

Design > Insert > Insert Sheet Columns

Design > Insert > Insert Sheet Columns


Solution:

Correct Answer:

A

Home > Insert > Insert Sheet Columns

होम > इन्सर्ट > इन्सर्ट शीट कॉलम्स (Home > Insert > Insert Sheet Columns)


Question 20


Which of the following fields will succinctly identify what we are writing about in an e-mail?

निम्नलिखित में से कौन सा फील्ड संक्षेप में यह पहचान लेगा कि हम ई-मेल में किस बारे में लिख रहे हैं?

Options

A

To

To

B

Subject

Subject

C

CC

CC

D

From

From


Solution:

Correct Answer:

B

Subject

सब्जेक्ट (Subject)


Question 21


Which of the following is true with reference to an e-mail address?

ई-मेल एड्रेस के संदर्भ में निम्न में से कौन सा कथन सत्य है?

Options

A

The # symbol is the connector between the domain and the person who the e-mail address belongs to.

The # symbol is the connector between the domain and the person who the e-mail address belongs to.

B

Popular e-mail providers for business or personal use include Gmail, Yahoo.

Popular e-mail providers for business or personal use include Gmail, Yahoo.

C

Domain names cannot be commercial or personal.

Domain names cannot be commercial or personal.

D

The domain name is not specific to any organisation.

The domain name is not specific to any organisation.


Solution:

Correct Answer:

B

Popular e-mail providers for business or personal use include Gmail, Yahoo.

व्यवसाय या व्यक्तिगत उपयोग के लिए लोकप्रिय ई-मेल प्रदाताओं में जीमेल, याहू शामिल हैं।


Question 22


openSUSE, Fedora, Debian etc., are versions of which of the following GUI based operating systems?

ओपनएसयूएसई (openSUSE), फेडोरा, डेबियन आदि, निम्नलिखित में से किस जीयूआई (GUI) आधारित ऑपरेटिंग सिस्टम का वर्ज़न हैं?

Options

A

MacOS X

MacOS X

B

Windows

Windows

C

Apple iOS

Apple iOS

D

Linux

Linux


Solution:

Correct Answer:

D

Linux

लिनक्स (Linux)


Question 23


Which of the following is example of application software?

निम्न में से कौन सा एप्लीकेशन सॉफ्टवेयर का उदाहरण है ?

Options

A

Device driver

Device driver

B

Email clients

Email clients

C

Boot program

Boot program

D

BIOS

BIOS


Solution:

Correct Answer:

B

Email clients

ईमेल क्लाइंट्स (Email clients)


Question 24


एमएस-एक्सेल 365 (MS-Excel 365) में निम्नलिखित में से कौन सा लॉजिकल फंक्शन TRUE रिटर्न करता है, यदि इसके सभी आर्ग्‍युमेंट TRUE हैं?

एमएस-एक्सेल 365 (MS-Excel 365) में निम्नलिखित में से कौन सा लॉजिकल फंक्शन TRUE रिटर्न करता है, यदि इसके सभी आर्ग्‍युमेंट TRUE हैं?

Options

A

ऑर (OR)

ऑर (OR)

B

एंड (AND)

एंड (AND)

C

फॉल्स (FALSE)

फॉल्स (FALSE)

D

नॉट (NOT)

नॉट (NOT)


Solution:

Correct Answer:

B

एंड (AND)

एंड (AND)


Question 25


Which of the following options is used to stop the footer from appearing on the title slide in MS-PowerPoint 365?

एमएस-पॉवर पॉइंट 365 (MS-PowerPoint 365) में फुटर को टाइटल स्लाइड पर प्रदर्शित होने से रोकने के लिए निम्नलिखित में से किस विकल्प का उपयोग किया जाता है?

Options

A

Don't show on title slide

Don't show on title slide

B

Show options

Show options

C

Show slides

Show slides

D

Don’t show on first slide

Don’t show on first slide


Solution:

Correct Answer:

A

Don't show on title slide

डोन्ट शो ऑन टाइटल स्लाइड (Don't show on title slide)


Question 26


Which of the following views allows to consistently format every slide, all at once, all in one place, and with one simple step on MS-PowerPoint?

निम्नलिखित में से कौन सा व्यू, एमएस-पॉवर पॉइंट (MS-PowerPoint) पर एक सिंपल स्टेप के साथ, सभी को एक ही स्थान पर, प्रत्येक स्लाइड को एक साथ लगातार फॉर्मेट करने की सुविधा देता है?

Options

A

Outline

Outline

B

Slide Master

Slide Master

C

Notes Page

Notes Page

D

Slide Sorter

Slide Sorter


Solution:

Correct Answer:

B

Slide Master

स्लाइड मास्टर (Slide Master)


Question 27


Which of the following MS-Word 365 features is used to raise one or more characters above the baseline?

निम्नलिखित में से कौन-सी एमएस-वर्ड 365 (MS-Word 365) के फीचर का उपयोग बेसलाइन से एक या अधिक कैरेक्टर्स (characters) को ऊपर उठाने के लिए किया जाता है?

Options

A

Subscript

Subscript

B

Superscript

Superscript

C

Word wrap

Word wrap

D

Drop cap

Drop cap


Solution:

Correct Answer:

B

Superscript

सुपरस्क्रिप्ट


Question 28


What will be the output of the formula “=COUNTA(A:A)”, if the contents of A1 to A8 are ‘DATA,1,2,3,4,5,6,COUNT’ in MS-Excel 365 ?

यदि एमएस-एक्‍सल (MS-Excel) 365 में A1 से A8 तक के कन्टेंट 'DATA,1,2,3,4,5,6,COUNT' हैं, तो फॉर्मूला "=COUNTA(A:A)" का आउटपुट क्या होगा?

Options

A

2

2

B

6

6

C

8

8

D

error

error


Solution:

Correct Answer:

C

8

8


Question 29


निम्न में से कौन सा एक वेब ब्राउज़र नहीं है?

निम्न में से कौन सा एक वेब ब्राउज़र नहीं है?

Options

A

यान्डेक्स (Yandex)

यान्डेक्स (Yandex)

B

ओपेरा (Opera)

ओपेरा (Opera)

C

सफारी (Safari)

सफारी (Safari)

D

नेटस्केप (Netscape)

नेटस्केप (Netscape)


Solution:

Correct Answer:

A

यान्डेक्स (Yandex)

यान्डेक्स (Yandex)


Question 30


Which of the following is a movable, concealable icon bar that is set on the very edge of the graphical user interface (GUI) desktop and serves as a launching pad for applications?

निम्नलिखित में से कौन सा मूवेबल, छुपाने योग्य आइकन बार है जो ग्राफिकल यूज़र इंटरफेस (GUI) डेस्कटॉप के बिल्कुल किनारे पर सेट है और एप्केलीकेशन्स के लिए लॉन्चिंग पैड के रूप में काम करता है?

Options

A

Task bar

Task bar

B

Start menu

Start menu

C

Icon

Icon

D

Application

Application


Solution:

Correct Answer:

A

Task bar

टास्क बार (Task bar)


Question 31


Which of the following function keys repeats the last command or action, if possible, in MS-Word 365?

एमएस-वर्ड 365 (MS-Word 365) में, निम्न में से कौन-सी फंक्शन कुंजी, यदि संभव हो तो, अंतिम कमांड या कार्य को दोहराती हैं?

Options

A

F4

F4

B

F1

F1

C

F2

F2

D

F3

F3


Solution:

Correct Answer:

A

F4

F4


Question 32


Which of the following is the information retrieval service provided by the internet?

निम्न में से कौन सी इंटरनेट द्वारा प्रदान की जाने वाली सूचना पुनर्प्राप्ति सेवा (information retrieval service) है?

Options

A

telnet

telnet

B

Gopher

Gopher

C

E-mail

E-mail

D

News group

News group


Solution:

Correct Answer:

B

Gopher

गोफर (Gopher)


Question 33


निम्नलिखित में से किसका उपयोग आईपी एड्रेस ( IP address) को एक नाम देने के लिए किया जाता है ताकि एक यूज़र इंटरनेट के सहयोग से एक नाम के द्वारा कंप्यूटर का पता लगा सके?

निम्नलिखित में से किसका उपयोग आईपी एड्रेस ( IP address) को एक नाम देने के लिए किया जाता है ताकि एक यूज़र इंटरनेट के सहयोग से एक नाम के द्वारा कंप्यूटर का पता लगा सके?

Options

A

टीसीपी/आईपी (TCP/IP)

टीसीपी/आईपी (TCP/IP)

B

एचटीटीपीएस (HTTPS)

एचटीटीपीएस (HTTPS)

C

डीएनएस (DNS)

डीएनएस (DNS)

D

यूडीपी (UDP)

यूडीपी (UDP)


Solution:

Correct Answer:

C

डीएनएस (DNS)

डीएनएस (DNS)


Question 34


Which of the following keyboard shortcuts is used to italicise text or remove italic formatting in MS-Excel 365?

एमएस-एक्सेल 365 (MS-Excel 365) में टेक्स्ट को इटैलिक करने या इटैलिक फॉर्मेटिंग को हटाने के लिए निम्न में से किस कीबोर्ड शॉर्टकट का उपयोग किया जाता है?

Options

A

Ctrl + 1

Ctrl + 1

B

Ctrl + 4

Ctrl + 4

C

Ctrl + 2

Ctrl + 2

D

Ctrl + 3

Ctrl + 3


Solution:

Correct Answer:

D

Ctrl + 3

Ctrl + 3


Question 35


डॉक्यूमेंट के लेआउट के संदर्भ में 'वाइड', 'मिरर्ड', 'नैरो' शब्द एमएस-वर्ड 365 (MS-Word 365) में निम्न में से किस विकल्प का वर्णन करता है?

डॉक्यूमेंट के लेआउट के संदर्भ में 'वाइड', 'मिरर्ड', 'नैरो' शब्द एमएस-वर्ड 365 (MS-Word 365) में निम्न में से किस विकल्प का वर्णन करता है?

Options

A

साइज़ (Size)

साइज़ (Size)

B

ब्रेक्स (Breaks)

ब्रेक्स (Breaks)

C

ऑरिएंटेशन (Orientation)

ऑरिएंटेशन (Orientation)

D

मार्जिन्स (Margins)

मार्जिन्स (Margins)


Solution:

Correct Answer:

D

मार्जिन्स (Margins)

मार्जिन्स (Margins)


Question 36


एमएस-वर्ड (MS-Word 365) में किसी डॉक्यूमेंट या कंटेनर के किनारे तक पहुँचने पर शब्द को तोड़ने के लिए निम्नलिखित में से कौन सी एक छोटी, डैश लाइन के उपयोग को संदर्भित करता है?

एमएस-वर्ड (MS-Word 365) में किसी डॉक्यूमेंट या कंटेनर के किनारे तक पहुँचने पर शब्द को तोड़ने के लिए निम्नलिखित में से कौन सी एक छोटी, डैश लाइन के उपयोग को संदर्भित करता है?

Options

A

रैप टेक्स्ट (Wrap text)

रैप टेक्स्ट (Wrap text)

B

हाईफनेशन (Hyphenation)

हाईफनेशन (Hyphenation)

C

मर्ज एंड सेंटर (Merge and Centre)

मर्ज एंड सेंटर (Merge and Centre)

D

स्ट्राइक थ्रू (Strikethrough)

स्ट्राइक थ्रू (Strikethrough)


Solution:

Correct Answer:

B

हाईफनेशन (Hyphenation)

हाईफनेशन (Hyphenation)


Question 37


Which of the following options is used to display the content of an active cell in MS-Excel 2019?

एमएस-एक्सेल 2019 (MS-Excel 2019) में एक्टिव सेल (active cell) की सामग्री को प्रदर्शित करने के लिए निम्नलिखित में से किस विकल्प का उपयोग किया जाता है?

Options

A

Title bar

Title bar

B

Scroll bar

Scroll bar

C

Formula bar

Formula bar

D

Ruler

Ruler


Solution:

Correct Answer:

C

Formula bar

फॉर्मूला बार (Formula bar)


Question 38


Which of the following search engines is launched by Microsoft?

निम्नलिखित में से कौन सा सर्च इंजन माइक्रोसॉफ्ट द्वारा लॉन्च किया गया है?

Options

A

Yandex

Yandex

B

Google

Google

C

Bing

Bing

D

Baidu

Baidu


Solution:

Correct Answer:

C

Bing

बिंग (Bing)


Question 39


Which of the following views of a MS-PowerPoint 365 presentation is used to edit the appearance of presentation handouts, including the layout, headers and footers, and background?

एमएस-पॉवरपॉइंट 365 (MS-PowerPoint 365) प्रेजेंटेशन का निम्नलिखित में से कौन सा व्यू लेआउट, हेडर और फुटर और बैकग्राउंड सहित प्रेजेंटेशन हैंडआउट्स की दिखावट को एडिट करने के लिए उपयोग किया जाता है?

Options

A

Handout Master

Handout Master

B

Slide Sorter

Slide Sorter

C

Slide Master

Slide Master

D

Notes Master

Notes Master


Solution:

Correct Answer:

A

Handout Master

हैंडआउट मास्टर (Handout Master)


Question 40


Which of the following best describes the terms Regular, Bold, and Italic in MS-Word 365?

निम्नलिखित में से कौन सा विकल्‍प एमएस-वर्ड 365 ( MS-Word 365) में रेगुलर, बोल्ड और इटैलिक शब्दों का सबसे अच्छा वर्णन करता है?

Options

A

Size

Size

B

Color

Color

C

Font Style

Font Style

D

Effects

Effects


Solution:

Correct Answer:

C

Font Style

फॉन्ट साइज़ (Font Size)


Question 41


Home shopping and advertising belong to which of the following categories of computer applications?

होम शॉपिंग और विज्ञापन, कंप्यूटर एप्लीकेशन की निम्नलिखित में से किस श्रेणी से संबंधित हैं?

Options

A

Healthcare

Healthcare

B

Education

Education

C

Marketing

Marketing

D

Military

Military


Solution:

Correct Answer:

C

Marketing

मार्केटिंग (Marketing)


Question 42


Who created Analytical Engine, one of the early computing devices?

प्रारंभिक कंप्यूटिंग डिवाइस में से एक, एनालिटिकल इंजन का निर्माण किसने किया था?

Options

A

Charles Babbage

Charles Babbage

B

Herman Hollerith

Herman Hollerith

C

John Napier 

John Napier 

D

Blaise Pascal

Blaise Pascal


Solution:

Correct Answer:

A

Charles Babbage

चार्ल्स बैबेज (Charles Babbage)


Question 43


How many options are available in the Alignment group for aligning text in a table in MS-Word 365?

एमएस-वर्ड 365 (MS-Word 365) में एक टेबल में टेक्स्ट अलाइन करने के लिए अलाइनमेंट ग्रुप में कितने विकल्प उपलब्ध होते हैं?

Options

A

6

6

B

7

7

C

3

3

D

9

9


Solution:

Correct Answer:

D

9

9


Question 44


Which of the following options in MS-Excel 365 refers to displaying only the rows that meet certain conditions?

एमएस-एक्सेल 365 (MS-Excel 365) में निम्न में से कौन सा विकल्प केवल कुछ शर्तों को पूरा करने वाली पंक्तियों को डिस्प्ले करने के लिए संदर्भित है?

Options

A

Pivoting

Pivoting

B

Filtering

Filtering

C

Merging

Merging

D

Sorting

Sorting


Solution:

Correct Answer:

B

Filtering

फिल्टरिंग (Filtering)


Question 45


Which of the following is the result of applying toggle case option to the word ‘Online Test’ in MS-Word 365?

निम्नलिखित में से कौन सा विकल्‍प एमएस-वर्ड 365 (MS-Word 365) में 'ऑनलाइन टेस्ट' शब्द पर टॉगल केस ऑप्शन लागू करने का परिणाम है?

Options

A

Online tesT

Online tesT

B

Online test

Online test

C

ONLINE TEST

ONLINE TEST

D

oNLINE tEST

oNLINE tEST


Solution:

Correct Answer:

D

oNLINE tEST

oNLINE tEST


Question 46


What is the shortcut key to start an MS-PowerPoint presentation from the current slide instead of the first slide?

एमएस-पॉवर पॉइंट (MS-PowerPoint) प्रेजेंटेशन को फर्स्ट स्लाइड के बजाय करंट स्लाइड से स्टार्ट करने के लिए शॉर्टकट कुंजी क्या है?

Options

A

Ctrl + F5

Ctrl + F5

B

Shift + F4

Shift + F4

C

Ctrl + F4

Ctrl + F4

D

Shift + F5

Shift + F5


Solution:

Correct Answer:

D

Shift + F5

Shift + F5


Question 47


Which of the following communication protocols is used to interconnect network devices on the internet?

निम्नलिखित में से कौन सा कम्‍युनिकेशन प्रोटोकॉल, इंटरनेट पर नेटवर्क डिवाइसेज़ को आपस में जोड़ने के लिए उपयोग किया जाता है?

Options

A

TCP/IP

TCP/IP

B

FTP

FTP

C

WWW

WWW

D

HTTP

HTTP


Solution:

Correct Answer:

A

TCP/IP

टीसीपी/आईपी (TCP/IP)


Question 48


Which of the following options of the Set Up Slide Show is used to select required slides for a presentation in MS-PowerPoint 365?

एमएस-पॉवर पॉइंट 365 (MS-PowerPoint 365) में प्रेजेंटेशन के लिए आवश्यक स्लाइड्स का चयन करने के लिए, सेट अप स्लाइड शो के निम्न में से कौन से विकल्प का उपयोग किया जाता है?

Options

A

Advance slides

Advance slides

B

Show slides

Show slides

C

Show type

Show type

D

Show options

Show options


Solution:

Correct Answer:

B

Show slides

शो स्लाइड्स (Show slides)


Question 49


Which of the following is the most recent version of the Windows operating system?

निम्न में से कौन सा विंडोज़ ऑपरेटिंग सिस्टम का नवीनतम संस्करण है?

Options

A

Windows 11

Windows 11

B

Windows 10

Windows 10

C

Windows Vista

Windows Vista

D

Windows 8

Windows 8


Solution:

Correct Answer:

A

Windows 11

विंडोज़ 11 (Windows 11)


Question 50


Which of the following is an example of volatile memory?

निम्न में से कौन सा वोलेटाइल मेमोरी का उदाहरण है?

Options

A

Hard drive

Hard drive

B

Flash memory

Flash memory

C

RAM

RAM

D

ROM

ROM


Solution:

Correct Answer:

C

RAM

रैम (RAM)


Question 51


सर्वाइकल (cervical) कैंसर से न‍िपटने के लिए भारत का पहला स्वदेशी रूप से विकसित वैक्‍सीन ______ को लॉन्च किया गया था।

सर्वाइकल (cervical) कैंसर से न‍िपटने के लिए भारत का पहला स्वदेशी रूप से विकसित वैक्‍सीन ______ को लॉन्च किया गया था।

Options

A

6 अप्रैल 2022

6 अप्रैल 2022

B

1 सितंबर 2022

1 सितंबर 2022

C

30 जनवरी 2022

30 जनवरी 2022

D

18 मार्च 2022

18 मार्च 2022


Solution:

Correct Answer:

B

1 सितंबर 2022

1 सितंबर 2022


Question 52


न्यू मूर द्वीप (New Moore Island) भारत और ______ के बीच विवाद का एक कारण था।

न्यू मूर द्वीप (New Moore Island) भारत और ______ के बीच विवाद का एक कारण था।

Options

A

पाकिस्तान

पाकिस्तान

B

म्यांमार

म्यांमार

C

श्रीलंका

श्रीलंका

D

बांग्लादेश

बांग्लादेश


Solution:

Correct Answer:

D

बांग्लादेश

बांग्लादेश


Question 53


अगस्त 2022 तक की जानकारी के अनुसार, मुख्य न्यायाधीश सहित सर्वोच्च न्यायालय में न्यायाधीशों की अधिकतम संख्या कितनी है?

अगस्त 2022 तक की जानकारी के अनुसार, मुख्य न्यायाधीश सहित सर्वोच्च न्यायालय में न्यायाधीशों की अधिकतम संख्या कितनी है?

Options

A

33

33

B

36

36

C

34

34

D

35

35


Solution:

Correct Answer:

C

34

34


Question 54


इनमें से क‍िसने जनवरी 2015 और अगस्त 2017 के बीच भारत सरकार के थिंक-टैंक, नीत‍ि (NITI) आयोग के उपाध्यक्ष के रूप में कार्य किया और एशियन डेवलपमेंट बैंक के पूर्व मुख्य अर्थशास्त्री हैं?

इनमें से क‍िसने जनवरी 2015 और अगस्त 2017 के बीच भारत सरकार के थिंक-टैंक, नीत‍ि (NITI) आयोग के उपाध्यक्ष के रूप में कार्य किया और एशियन डेवलपमेंट बैंक के पूर्व मुख्य अर्थशास्त्री हैं?

Options

A

अविनाश दीक्षित

अविनाश दीक्षित

B

सुमन बेरी

सुमन बेरी

C

बिबेक देबरॉय

बिबेक देबरॉय

D

अरविंद पनगढ़िया

अरविंद पनगढ़िया


Solution:

Correct Answer:

D

अरविंद पनगढ़िया

अरविंद पनगढ़िया


Question 55


अगस्त 2022 तक की जानकारी के अनुसार, निम्नलिखित में से कौन सा विकल्प क्रमशः राज्य सभा की सदस्‍य संख्‍या और राज्य सभा की सदस्‍य संख्या की संवैधानिक सीमा को सही ढंग से दर्शाता है?

अगस्त 2022 तक की जानकारी के अनुसार, निम्नलिखित में से कौन सा विकल्प क्रमशः राज्य सभा की सदस्‍य संख्‍या और राज्य सभा की सदस्‍य संख्या की संवैधानिक सीमा को सही ढंग से दर्शाता है?

Options

A

238 और 234

238 और 234

B

245 और 250

245 और 250

C

238 और 250

238 और 250

D

235 और 250

235 और 250


Solution:

Correct Answer:

B

245 और 250

245 और 250


Question 56


Which of the following airports is located in Qatar?

निम्नलिखित में से कौन सा हवाई अड्डा क़तर में स्थित है?

Options

A

Dallas Fort International Airport

Dallas Fort International Airport

B

Hamad International Airport

Hamad International Airport

C

Charlotte Douglas International Airport

Charlotte Douglas International Airport

D

Heathrow International Airport

Heathrow International Airport


Solution:

Correct Answer:

B

Hamad International Airport

हमद अंतर्राष्ट्रीय हवाई अड्डा


Question 57


Under which Indian constitutional amendment act the urban local bodies were granted constitutional status?

किस भारतीय संविधान संशोधन अधिनियम के तहत शहरी स्थानीय निकायों को संवैधानिक दर्जा दिया गया था?

Options

A

92nd Amendment Act, 2003

92nd Amendment Act, 2003

B

86th Amendment Act, 2002

86th Amendment Act, 2002

C

88th Amendment Act, 2003

88th Amendment Act, 2003

D

74th Amendment Act, 1992

74th Amendment Act, 1992


Solution:

Correct Answer:

D

74th Amendment Act, 1992

74वां संशोधन अधिनियम, 1992


Question 58


अगस्त 2022 में 'स्मार्टबॉक्सर' नामक एनालिटिक्स प्लेटफॉर्म विकसित करने के लिए निम्नलिखित में से किस संस्थान के शोधकर्ताओं ने इंस्पायर इंस्टीट्यूट ऑफ स्पोर्ट्स, कर्नाटक के साथ सहभाग‍िता की?

अगस्त 2022 में 'स्मार्टबॉक्सर' नामक एनालिटिक्स प्लेटफॉर्म विकसित करने के लिए निम्नलिखित में से किस संस्थान के शोधकर्ताओं ने इंस्पायर इंस्टीट्यूट ऑफ स्पोर्ट्स, कर्नाटक के साथ सहभाग‍िता की?

Options

A

भारतीय प्रौद्योगिकी संस्थान, खड़गपुर

भारतीय प्रौद्योगिकी संस्थान, खड़गपुर

B

भारतीय प्रौद्योगिकी संस्थान, मद्रास

भारतीय प्रौद्योगिकी संस्थान, मद्रास

C

भारतीय प्रौद्योगिकी संस्थान, दिल्ली

भारतीय प्रौद्योगिकी संस्थान, दिल्ली

D

भारतीय प्रौद्योगिकी संस्थान, बॉम्बे

भारतीय प्रौद्योगिकी संस्थान, बॉम्बे


Solution:

Correct Answer:

B

भारतीय प्रौद्योगिकी संस्थान, मद्रास

भारतीय प्रौद्योगिकी संस्थान, मद्रास


Question 59


कृष्णा बेसिन ______ तक फैली हुई है, जिसका कुल क्षेत्रफल 2,58,948 km2 है, जो देश के कुल भौगोलिक क्षेत्र का लगभग 8% है।

कृष्णा बेसिन ______ तक फैली हुई है, जिसका कुल क्षेत्रफल 2,58,948 km2 है, जो देश के कुल भौगोलिक क्षेत्र का लगभग 8% है।

Options

A

आंध्र प्रदेश, तेलंगाना और कर्नाटक

आंध्र प्रदेश, तेलंगाना और कर्नाटक

B

आंध्र प्रदेश, महाराष्ट्र और कर्नाटक

आंध्र प्रदेश, महाराष्ट्र और कर्नाटक

C

कर्नाटक, केरल और तमिलनाडु

कर्नाटक, केरल और तमिलनाडु

D

महाराष्ट्र, कर्नाटक और केरल

महाराष्ट्र, कर्नाटक और केरल


Solution:

Correct Answer:

B

आंध्र प्रदेश, महाराष्ट्र और कर्नाटक

आंध्र प्रदेश, महाराष्ट्र और कर्नाटक


Question 60


निम्नलिखित में से किसने लखनऊ में 1857 के विद्रोह का नेतृत्व किया?

निम्नलिखित में से किसने लखनऊ में 1857 के विद्रोह का नेतृत्व किया?

Options

A

नाना साहब पेशवा

नाना साहब पेशवा

B

मौलवी लियाकत अली

मौलवी लियाकत अली

C

कुंवर सिंह

कुंवर सिंह

D

बेगम हजरत महल

बेगम हजरत महल


Solution:

Correct Answer:

D

बेगम हजरत महल

बेगम हजरत महल


Question 61


चालुक्य वंश के राजाओं द्वारा निर्मित लाड खान मंदिर (Lad Khan Temple) कहां स्थित है?

चालुक्य वंश के राजाओं द्वारा निर्मित लाड खान मंदिर (Lad Khan Temple) कहां स्थित है?

Options

A

चिदंबरम

चिदंबरम

B

ऐहोले

ऐहोले

C

महाबलीपुरम

महाबलीपुरम

D

भभुआ

भभुआ


Solution:

Correct Answer:

B

ऐहोले

ऐहोले


Question 62


अगस्त 2022 तक, निम्नलिखित में से कौन से देशबहुक्षेत्रीय तकनीकी और आर्थिक सहयोग के लिए बंगाल की खाड़ी पहल (BIMSTEC) का हिस्सा नहीं हैं?

अगस्त 2022 तक, निम्नलिखित में से कौन से देशबहुक्षेत्रीय तकनीकी और आर्थिक सहयोग के लिए बंगाल की खाड़ी पहल (BIMSTEC) का हिस्सा नहीं हैं?

Options

A

बांग्लादेश और भूटान

बांग्लादेश और भूटान

B

श्रीलंका और थाईलैंड

श्रीलंका और थाईलैंड

C

म्यांमार और नेपाल

म्यांमार और नेपाल

D

पाकिस्तान और ईरान

पाकिस्तान और ईरान


Solution:

Correct Answer:

D

पाकिस्तान और ईरान

पाकिस्तान और ईरान


Question 63


इनमें से कौन सी भाषाएं भारत के संविधान की आठवीं अनुसूची में क्रमशः 2004 और 1992 में शामिल की गई थीं?

इनमें से कौन सी भाषाएं भारत के संविधान की आठवीं अनुसूची में क्रमशः 2004 और 1992 में शामिल की गई थीं?

Options

A

डोगरी और संथाली

डोगरी और संथाली

B

कोंकणी और मणिपुरी

कोंकणी और मणिपुरी

C

बोडो और नेपाली

बोडो और नेपाली

D

सिंधी और मैथिली

सिंधी और मैथिली


Solution:

Correct Answer:

C

बोडो और नेपाली

बोडो और नेपाली


Question 64


तख्त श्री केशगढ़ साहिब पंजाब के आनंदपुर साहिब में स्थित है, जो खालसा का सृजन स्‍थल है। इसकी स्थापना ______ द्वारा की गई थी।

तख्त श्री केशगढ़ साहिब पंजाब के आनंदपुर साहिब में स्थित है, जो खालसा का सृजन स्‍थल है। इसकी स्थापना ______ द्वारा की गई थी।

Options

A

श्री गुरू हरगोबिंद जी

श्री गुरू हरगोबिंद जी

B

श्री गुरू हरकिशन जी

श्री गुरू हरकिशन जी

C

श्री गुरु गोबिंद सिंह जी

श्री गुरु गोबिंद सिंह जी

D

श्री गुरु तेग बहादुर जी

श्री गुरु तेग बहादुर जी


Solution:

Correct Answer:

C

श्री गुरु गोबिंद सिंह जी

श्री गुरु गोबिंद सिंह जी


Question 65


अराव‍िडु वंश (Aravidu dynasty), दक्षिणी भारत में विजयनगर के हिंदू साम्राज्य का चौथा और अंतिम वंश जो ______ द्वारा स्थापित किया गया था।

अराव‍िडु वंश (Aravidu dynasty), दक्षिणी भारत में विजयनगर के हिंदू साम्राज्य का चौथा और अंतिम वंश जो ______ द्वारा स्थापित किया गया था।

Options

A

श्रीरंग देव राय (Sriranga Deva Raya)

श्रीरंग देव राय (Sriranga Deva Raya)

B

तिरुमल देव राय (Tirumala Deva Raya)

तिरुमल देव राय (Tirumala Deva Raya)

C

रामदेव राय (Rama Deva Raya)

रामदेव राय (Rama Deva Raya)

D

पेड वेंटक राय (Peda Venkata Raya)

पेड वेंटक राय (Peda Venkata Raya)


Solution:

Correct Answer:

B

तिरुमल देव राय (Tirumala Deva Raya)

तिरुमल देव राय (Tirumala Deva Raya)


Question 66


In August 2022, the RBI increased the policy repo rate under the Liquidity Adjustment Facility (LAF) by 50 basis points to:

अगस्त 2022 में, आरबीआई (RBI) नेचलनिधि समायोजन सुविधा ( Liquidity Adjustment Facility ) के तहत पॉलिसी रेपो दर को 50 आधार अंक बढ़ाकर ______ कर दिया।

Options

A

5.40%

5.40%

B

4.95%

4.95%

C

5.65%

5.65%

D

5.15%

5.15%


Solution:

Correct Answer:

A

5.40%

5.40%


Question 67


भारत का पहला 'डार्क-स्काई रिजर्व', जिसे नाइट स्काई सैंक्चुअरी के नाम से भी जाना जाता है, कहां स्थापित किया जाएगा?

भारत का पहला 'डार्क-स्काई रिजर्व', जिसे नाइट स्काई सैंक्चुअरी के नाम से भी जाना जाता है, कहां स्थापित किया जाएगा?

Options

A

पंजाब

पंजाब

B

दिल्ली

दिल्ली

C

राजस्थान

राजस्थान

D

लद्दाख

लद्दाख


Solution:

Correct Answer:

D

लद्दाख

लद्दाख


Question 68


AD 1704 में, औरंगजेब ने फिर से तोरणा किले पर कब्जा कर लिया और इसे '______ ' नाम दिया।

AD 1704 में, औरंगजेब ने फिर से तोरणा किले पर कब्जा कर लिया और इसे '______ ' नाम दिया।

Options

A

फतेहाबाद (Faathabad)

फतेहाबाद (Faathabad)

B

फैजागढ़ (Faijagarh)

फैजागढ़ (Faijagarh)

C

फ़ुतुलगैब (Futulgaib)

फ़ुतुलगैब (Futulgaib)

D

फरीनगढ़ (Fareengarh)

फरीनगढ़ (Fareengarh)


Solution:

Correct Answer:

C

फ़ुतुलगैब (Futulgaib)

फ़ुतुलगैब (Futulgaib)


Question 69


सिन्धु घाटी सभ्यता के निम्नलिखित में से किस स्थल में शतरंज की बिसात जैसी संरचना (chessboard-like structure) पाई गई थी?

सिन्धु घाटी सभ्यता के निम्नलिखित में से किस स्थल में शतरंज की बिसात जैसी संरचना (chessboard-like structure) पाई गई थी?

Options

A

बनावली

बनावली

B

मोहनजोदड़ो

मोहनजोदड़ो

C

लोथल

लोथल

D

धोलावीरा

धोलावीरा


Solution:

Correct Answer:

C

लोथल

लोथल


Question 70


निम्नलिखित में से कौन सी पंचवर्षीय योजनाएं क्रमशः गाडगिल सूत्र और महालनोबिस मॉडल पर आधारित हैं?

निम्नलिखित में से कौन सी पंचवर्षीय योजनाएं क्रमशः गाडगिल सूत्र और महालनोबिस मॉडल पर आधारित हैं?

Options

A

पांचवीं पंचवर्षीय योजना और दूसरी पंचवर्षीय योजना

पांचवीं पंचवर्षीय योजना और दूसरी पंचवर्षीय योजना

B

पांचवीं पंचवर्षीय योजना और तीसरी पंचवर्षीय योजना

पांचवीं पंचवर्षीय योजना और तीसरी पंचवर्षीय योजना

C

चौथी पंचवर्षीय योजना और दूसरी पंचवर्षीय योजना

चौथी पंचवर्षीय योजना और दूसरी पंचवर्षीय योजना

D

छठी पंचवर्षीय योजना और तीसरी पंचवर्षीय योजना

छठी पंचवर्षीय योजना और तीसरी पंचवर्षीय योजना


Solution:

Correct Answer:

C

चौथी पंचवर्षीय योजना और दूसरी पंचवर्षीय योजना

चौथी पंचवर्षीय योजना और दूसरी पंचवर्षीय योजना


Question 71


जून 2022 में, गेयटी थिएटर (Gaiety Theatre) में 'उनमेश' नामक तीन दिवसीय अंतर्राष्ट्रीय साहित्यिक उत्सव भारत के किस राज्य में आयोजित किया गया था?

जून 2022 में, गेयटी थिएटर (Gaiety Theatre) में 'उनमेश' नामक तीन दिवसीय अंतर्राष्ट्रीय साहित्यिक उत्सव भारत के किस राज्य में आयोजित किया गया था?

Options

A

उत्तराखंड

उत्तराखंड

B

उत्तर प्रदेश

उत्तर प्रदेश

C

बिहार

बिहार

D

हिमाचल प्रदेश

हिमाचल प्रदेश


Solution:

Correct Answer:

D

हिमाचल प्रदेश

हिमाचल प्रदेश


Question 72


भारतीय संविधान का निम्नलिखित में से कौन सा अनुच्छेद राज्यपाल की विशेष मामलों में दंड क्षमा करने आद‍ि और दंडादेश में निलंबन, परिहार या लघुकरण की शक्‍त‍ियों से संबंधित है?

भारतीय संविधान का निम्नलिखित में से कौन सा अनुच्छेद राज्यपाल की विशेष मामलों में दंड क्षमा करने आद‍ि और दंडादेश में निलंबन, परिहार या लघुकरण की शक्‍त‍ियों से संबंधित है?

Options

A

अनुच्छेद 161

अनुच्छेद 161

B

अनुच्छेद 143

अनुच्छेद 143

C

अनुच्छेद 124

अनुच्छेद 124

D

अनुच्छेद 104

अनुच्छेद 104


Solution:

Correct Answer:

A

अनुच्छेद 161

अनुच्छेद 161


Question 73


निम्नलिखित में से किस देश ने मई 2022 में 7वीं ब्रिक्स (BRICS) संस्कृति मंत्रियों की बैठक की मेजबानी की?

निम्नलिखित में से किस देश ने मई 2022 में 7वीं ब्रिक्स (BRICS) संस्कृति मंत्रियों की बैठक की मेजबानी की?

Options

A

फ्रांस

फ्रांस

B

भारत

भारत

C

रूस

रूस

D

चीन

चीन


Solution:

Correct Answer:

D

चीन

चीन


Question 74


इनमें से कौन खेल अनुभव और यात्रा प्‍लेटफॉर्म ड्रीमसेटगो (DreamSetGo) का पहला ब्रांड एंबेसडर है?

इनमें से कौन खेल अनुभव और यात्रा प्‍लेटफॉर्म ड्रीमसेटगो (DreamSetGo) का पहला ब्रांड एंबेसडर है?

Options

A

राहुल द्रविड़

राहुल द्रविड़

B

विराट कोहली

विराट कोहली

C

सचिन तेंदुलकर

सचिन तेंदुलकर

D

सौरव गांगुली

सौरव गांगुली


Solution:

Correct Answer:

D

सौरव गांगुली

सौरव गांगुली


Question 75


The total value of all the finished goods and services produced by a country’s citizens in a given financial year, irrespective of their location, is called:

किसी दिए गए वित्तीय वर्ष में किसी देश के नागरिकों द्वारा उत्पादित सभी तैयार वस्तुओं और सेवाओं का कुल मूल्य, चाहे उनका स्थान कुछ भी हो,______ कहलाता है।

Options

A

Net Domestic Product

Net Domestic Product

B

Gross Domestic Product

Gross Domestic Product

C

Net National Product

Net National Product

D

Gross National Product

Gross National Product


Solution:

Correct Answer:

D

Gross National Product

सकल राष्ट्रीय उत्पाद (Gross National Product)


Question 76


Select the number from among the given options that can replace the question mark (?) in the following series.
1735, 1752, 1725, 1762, ?

दिए गए विकल्पों में से उस संख्या का चयन कीजिए, जो निम्नलिखित श्रृंखला में प्रश्नवाचक चिह्न (?) को प्रतिस्थापित कर सकती है।
1735, 1752, 1725, 1762, ?

Options

A

1517

1517

B

1727

1727

C

1751

1751

D

1715

1715


Solution:

Correct Answer:

D

1715

1715


Question 77


Four words have been given, out of which three are alike in some manner and one is different. Select the word that is different.

चार शब्द दिए गए हैं, जिनमें से तीन किसी न किसी तरीके से एक समान हैं, और एक असंगत है। उस असंगत शब्द का चयन कीजिए।

Options

A

Bray

Bray

B

Cackle

Cackle

C

Trot

Trot

D

Croak

Croak


Solution:

Correct Answer:

C

Trot

दुलकी चाल (Trot)


Question 78


Select the option that is related to the third number in the same way as the second number is related to the first number.
64 : 36 :: 84 : ?

उस विकल्प का चयन कीजिए जो तीसरी संख्या से ठीक उसी प्रकार संबंधित है, जिस प्रकार दूसरी संख्या पहली संख्या से संबंधित है।
64 : 36 :: 84 : ?

Options

A

58

58

B

64

64

C

44

44

D

46

46


Solution:

Correct Answer:

D

46

46


Question 79


Four figures have been given, out of which three are alike in some manner and one is different. Select the figure that is different.

चार आकृतियां दी गई हैं, जिनमें से तीन किसी न किसी तरीके से एक समान हैं, और एक असंगत है। उस असंगत आकृति का चयन कीजिए।

Options

A

1

1

B

3

3

C

4

4

D

2

2


Solution:

Correct Answer:

B

3

3


Question 80


Select the letter-cluster from among the given options that can replace the question mark (?) in the following series.
LQ, PT, TW, ?, BC

दिए गए विकल्पों में से उस अक्षर-समूह का चयन कीजिए, जो निम्नलिखित श्रृंखला में प्रश्नवाचक चिह्न (?) को प्रतिस्थापित कर सके।
LQ, PT, TW, ?, BC

Options

A

YY

YY

B

YX

YX

C

XY

XY

D

XZ

XZ


Solution:

Correct Answer:

D

XZ

XZ


Question 81


Select the number from among the given options that can replace the question mark (?) in the following series.
2163, 2085, 2010, 1938, ?

दिए गए विकल्पों में से उस संख्या का चयन कीजिए, जो निम्नलिखित श्रृंखला में प्रश्नवाचक चिह्न (?) को प्रतिस्थापित कर सकती है।
2163, 2085, 2010, 1938, ?

Options

A

1866

1866

B

1869

1869

C

1863

1863

D

1860

1860


Solution:

Correct Answer:

B

1869

1869


Question 82


Sachin is the only son of Maya. Dev is the son of Mayank. Devika is the daughter of Dev. Sachin is the brother of Devika. How is Maya related to Mayank?

सचिन, माया का इकलौता पुत्र है। देव, मयंक का पुत्र है। देविका, देव की पुत्री है। सचिन, देविका का भाई है। माया का मयंक से क्या संबंध है?

Options

A

Wife

Wife

B

Daughter

Daughter

C

Daughter-in-law

Daughter-in-law

D

Mother-in-law

Mother-in-law


Solution:

Correct Answer:

C

Daughter-in-law

पुत्र-वधू


Question 83


In a certain code language, ‘RAT’ is coded as ‘126’. How will ‘GOD’ be coded in that language?

एक निश्चित कूट भाषा में, 'RAT' को '126' के रूप में कोडित किया जाता है। उसी भाषा में 'GOD' को किस रूप में कोडित किया जाएगा?

Options

A

78

78

B

81

81

C

79

79

D

165

165


Solution:

Correct Answer:

D

165

165


Question 84


उस विकल्प का चयन कीजिए जो तीसरे शब्द से ठीक उसी प्रकार संबंधित है, जिस प्रकार दूसरा शब्द पहले शब्द से संबंधित है।
तिरुवनंतपुरम : केरल :: गंगटोक : ?

उस विकल्प का चयन कीजिए जो तीसरे शब्द से ठीक उसी प्रकार संबंधित है, जिस प्रकार दूसरा शब्द पहले शब्द से संबंधित है।
तिरुवनंतपुरम : केरल :: गंगटोक : ?

Options

A

सिक्किम

सिक्किम

B

नागालैंड

नागालैंड

C

आंध्र प्रदेश

आंध्र प्रदेश

D

मेघालय

मेघालय


Solution:

Correct Answer:

A

सिक्किम

सिक्किम


Question 85


Select the answer figure that is related to problem figure C in the same way as problem figure B is related to problem figure A.

उस उत्तर-आकृति का चयन कीजिए जो प्रश्न-आकृति C से उसी प्रकार संबंधित है जिस प्रकार प्रश्न-आकृति B, प्रश्न-आकृति A से संबंधित है।

Options

A

3

3

B

2

2

C

1

1

D

4

4


Solution:

Correct Answer:

A

3

3


Question 86


Select the option that is related to the third term in the same way as the second term is related to the first term.
CURTAIN : NIATCUR :: GARMENT : ?

उस विकल्प का चयन कीजिए जो तीसरे पद से ठीक उसी प्रकार संबंधित है, जिस प्रकार दूसरा पद पहले पद से संबंधित है।
CURTAIN : NIATCUR :: GARMENT : ?

Options

A

TNEMGAR

TNEMGAR

B

TNMEGAR

TNMEGAR

C

TENMGAR

TENMGAR

D

TNEMGRA

TNEMGRA


Solution:

Correct Answer:

A

TNEMGAR

TNEMGAR


Question 87


Four number-triads have been given, out of which three are alike in some manner and one is different. Select the number-triad that is different.

चार संख्या-त्रिक दिए गए हैं, जिनमें से तीन किसी न किसी तरीके से एक समान हैं और एक असंगत है। उस असंगत संख्या-त्रिक का चयन कीजिए।

Options

A

6, 30, 72

6, 30, 72

B

9, 45, 110

9, 45, 110

C

8, 40, 96

8, 40, 96

D

3, 15, 36

3, 15, 36


Solution:

Correct Answer:

B

9, 45, 110

9, 45, 110


Question 88


उस शब्द का चयन कीजिए जिसे दी गई कड़ी वाले अक्षरों का उपयोग करके केवल उतनी बार बनाया जा सकता है, जितनी बार इस कड़ी में उन अक्षरों का उपयोग किया गया है।
NRAOCBHJWKFSLPEUT

उस शब्द का चयन कीजिए जिसे दी गई कड़ी वाले अक्षरों का उपयोग करके केवल उतनी बार बनाया जा सकता है, जितनी बार इस कड़ी में उन अक्षरों का उपयोग किया गया है।
NRAOCBHJWKFSLPEUT

Options

A

TOPPER

TOPPER

B

INPUT

INPUT

C

WORKED

WORKED

D

FOREST

FOREST


Solution:

Correct Answer:

D

FOREST

FOREST


Question 89


Four figures have been given, out of which three are alike in some manner and one is different. Select the figure that is different.

चार आकृतियां दी गई हैं, जिनमें से तीन किसी न किसी तरीके से एक समान हैं, और एक असंगत है। उस असंगत आकृति का चयन कीजिए।

Options

A

2

2

B

1

1

C

4

4

D

3

3


Solution:

Correct Answer:

A

2

2


Question 90


चार शब्द दिए गए हैं, जिनमें से तीन किसी न किसी तरीके से एक समान हैं, और एक असंगत है। उस असंगत शब्द का चयन कीजिए।

चार शब्द दिए गए हैं, जिनमें से तीन किसी न किसी तरीके से एक समान हैं, और एक असंगत है। उस असंगत शब्द का चयन कीजिए।

Options

A

योद्धा

योद्धा

B

अभिनेता

अभिनेता

C

न्यायालय

न्यायालय

D

लिपिक

लिपिक


Solution:

Correct Answer:

C

न्यायालय

न्यायालय


Question 91


Four figures have been given, out of which three are alike in some manner and one is different. Select the figure that is different.

चार आकृतियां दी गई हैं, जिनमें से तीन किसी न किसी तरीके से एक समान हैं, और एक असंगत है। उस असंगत आकृति का चयन कीजिए।

Options

A

C

C

B

D

D

C

A

A

D

B

B


Solution:

Correct Answer:

A

C

C


Question 92


In a certain code language, ‘HORSE’ is written as ‘I15ST5’. How will ‘EAGLE’ be written in that language?

किसी निश्चित कूट भाषा में, 'HORSE' को 'I15ST5' लिखा जाता है। उसी भाषा में 'EAGLE' को कैसे लिखा जाएगा?

Options

A

G1HM5

G1HM5

B

51HMF

51HMF

C

52HM5

52HM5

D

51HM5

51HM5


Solution:

Correct Answer:

D

51HM5

51HM5


Question 93


Four letter-clusters have been given, out of which three are alike in some manner and one is different. Select the letter-cluster that is different.

चार अक्षर-समूह दिए गए हैं, जिनमें से तीन किसी न किसी तरीके से एक समान हैं, और एक असंगत है। उस असंगत अक्षर-समूह का चयन कीजिए।

Options

A

LOR

LOR

B

EHK

EHK

C

WZC

WZC

D

UXB

UXB


Solution:

Correct Answer:

D

UXB

UXB


Question 94


Four letter-cluster triads have been given, out of which three are alike in some manner and one is different. Select the letter-cluster triad that is different.

चार अक्षर-समूह वाले त्रिक दिए गए हैं, जिनमें से तीन किसी न किसी तरीके से एक समान हैं, और एक असंगत है। उस असंगत अक्षर-समूह त्रिक का चयन कीजिए।

Options

A

ELS – SLE – JQX

ELS – SLE – JQX

B

INS – SNI – NSX

INS – SNI – NSX

C

AGM – GMA – FRL

AGM – GMA – FRL

D

DHL – LHD – IMQ

DHL – LHD – IMQ


Solution:

Correct Answer:

C

AGM – GMA – FRL

AGM – GMA – FRL


Question 95


Four number-pairs have been given, out of which three are alike in some manner and one is different. Select the number-pair that is different.

चार संख्या-युग्म दिए गए हैं, जिनमें से तीन किसी न किसी तरीके से एक समान हैं और एक असंगत है। उस असंगत संख्या-युग्म का चयन कीजिए।

Options

A

17-51

17-51

B

29-87

29-87

C

39-117

39-117

D

43-129

43-129


Solution:

Correct Answer:

C

39-117

39-117


Question 96


निखिल, चारुल का भाई है और चारुल की दादी नेहा हैं। यदि अलका का विवाह नेहा के इकलौते बेटे से हुआ है, तो अलका का नेहा से क्या संबंध है?

निखिल, चारुल का भाई है और चारुल की दादी नेहा हैं। यदि अलका का विवाह नेहा के इकलौते बेटे से हुआ है, तो अलका का नेहा से क्या संबंध है?

Options

A

बहू

बहू

B

सास

सास

C

बेटी

बेटी

D

माता

माता


Solution:

Correct Answer:

A

बहू

बहू


Question 97


Select the option that is related to the third number in the same way as the second number is related to the first number.
56 : 196 :: 84 : ?

उस विकल्प का चयन कीजिए जो तीसरी संख्या से ठीक उसी प्रकार संबंधित है, जिस प्रकार दूसरी संख्या पहली संख्या से संबंधित है।
56 : 196 :: 84 : ?

Options

A

429

429

B

294

294

C

492

492

D

249

249


Solution:

Correct Answer:

B

294

294


Question 98


यदि Q का अर्थ 'जोड़ना' है, J का अर्थ 'गुणा करना' है, T का अर्थ 'घटाना' है, और K का अर्थ 'भाग देना है', तो निम्नलिखित व्यंजक का मान कितना होगा?
45 T 56 Q 78 J 9 K 3

यदि Q का अर्थ 'जोड़ना' है, J का अर्थ 'गुणा करना' है, T का अर्थ 'घटाना' है, और K का अर्थ 'भाग देना है', तो निम्नलिखित व्यंजक का मान कितना होगा?
45 T 56 Q 78 J 9 K 3

Options

A

232

232

B

127

127

C

223

223

D

15

15


Solution:

Correct Answer:

C

223

223


Question 99


In a certain code language, ‘DOOR IS OPEN’ is coded as ‘Ju Kp Sa’, and ‘THERE IS DOOR’ is coded as ‘Ni Ju Kp’. What is the code for ‘OPEN’?

किसी निश्चित कूट भाषा में, ‘DOOR IS OPEN’ को ‘Ju Kp Sa’ के रूप में कूटबद्ध किया जाता है और ‘THERE IS DOOR’ को ‘Ni Ju Kp’ के रूप में कूटबद्ध किया जाता है। ‘OPEN’ को किस रूप में कूटबद्ध किया जाएगा?

Options

A

Ni

Ni

B

Kp

Kp

C

Ju

Ju

D

Sa

Sa


Solution:

Correct Answer:

D

Sa

Sa


Question 100


Select the option that is related to the fifth number in the same way as the second number is related to the first number and the fourth number is related to the third number.
15 : 69 :: 29 : 133.4 :: 57 : ?

उस विकल्प का चयन कीजिए जो पांचवीं संख्या से ठीक उसी प्रकार संबंधित है, जिस प्रकार दूसरी संख्या पहली संख्या से संबंधित है और चौथी संख्या तीसरी संख्या से संबंधित है।
15 : 69 :: 29 : 133.4 :: 57 : ?

Options

A

269.4

269.4

B

226.2

226.2

C

262.2

262.2

D

222.6

222.6


Solution:

Correct Answer:

C

262.2

262.2


Question 101


चार शब्द-त्रिक दिए गए हैं, जिनमें से तीन किसी न किसी तरीके से एक समान हैं, और एक असंगत है। उस असंगत शब्द-त्रिक का चयन कीजिए।

चार शब्द-त्रिक दिए गए हैं, जिनमें से तीन किसी न किसी तरीके से एक समान हैं, और एक असंगत है। उस असंगत शब्द-त्रिक का चयन कीजिए।

Options

A

आकर्षक – सुंदर - मनोहर

आकर्षक – सुंदर - मनोहर

B

धोखा देना – ठगना – छलना

धोखा देना – ठगना – छलना

C

आरोहण – चढ़ाई – दुर्गम पदयात्रा

आरोहण – चढ़ाई – दुर्गम पदयात्रा

D

सहायता – सहयोग – बाधा

सहायता – सहयोग – बाधा


Solution:

Correct Answer:

D

सहायता – सहयोग – बाधा

सहायता – सहयोग – बाधा


Question 102


Select the number from among the given options that can replace the question mark (?) in the following series.
1845, 1942, 2039, 2136, ?

दिए गए विकल्पों में से उस संख्या का चयन कीजिए, जो निम्नलिखित श्रृंखला में प्रश्नवाचक चिह्न (?) को प्रतिस्थापित कर सकती ह।
1845, 1942, 2039, 2136, ?

Options

A

2323

2323

B

2233

2233

C

2332

2332

D

2328

2328


Solution:

Correct Answer:

B

2233

2233


Question 103


Four figures have been given, out of which three are alike in some manner and one is different. Select the figure that is different.

चार आकृतियां दी गई हैं, जिनमें से तीन किसी न किसी तरीके से एक समान हैं, और एक असंगत है। उस असंगत आकृति का चयन कीजिए।

Options

A

2

2

B

3

3

C

1

1

D

4

4


Solution:

Correct Answer:

B

3

3


Question 104


Select the alphanumeric-cluster from among the given options that can replace the question mark (?) in the following series.
57T, 70Z, 85F, 102L, ?

दिए गए विकल्पों में से उस अक्षरांकीय-समूह का चयन कीजिए, जो निम्नलिखित श्रृंखला में प्रश्न चिह्न (?) को प्रतिस्थापित कर सके।
57T, 70Z, 85F, 102L, ?

Options

A

108T

108T

B

121R

121R

C

112S

112S

D

211R

211R


Solution:

Correct Answer:

B

121R

121R


Question 105


यदि नीचे दिए गए अक्षरों को अंग्रेज़ी वर्णमाला श्रृंखला में उनके स्थान के आरोही क्रम में बाएं से दाएं व्यवस्थित किया जाना हो,, तो बाएं से 5वें, 10वें, 12वें, 13वें, 14वें और 20वें अक्षर का उपयोग करके दिए गए विकल्पों में से कौन-सा अर्थपूर्ण शब्द बनाया जा सकता है? सभी अक्षरों का उपयोग किया जाना चाहिए और प्रत्येक अक्षर का उपयोग केवल एक बार ही किया जाना है।
PCVZQNAHKMUEBYIJRLGDO

यदि नीचे दिए गए अक्षरों को अंग्रेज़ी वर्णमाला श्रृंखला में उनके स्थान के आरोही क्रम में बाएं से दाएं व्यवस्थित किया जाना हो,, तो बाएं से 5वें, 10वें, 12वें, 13वें, 14वें और 20वें अक्षर का उपयोग करके दिए गए विकल्पों में से कौन-सा अर्थपूर्ण शब्द बनाया जा सकता है? सभी अक्षरों का उपयोग किया जाना चाहिए और प्रत्येक अक्षर का उपयोग केवल एक बार ही किया जाना है।
PCVZQNAHKMUEBYIJRLGDO

Options

A

DONKEY

DONKEY

B

MONKEY

MONKEY

C

MONEY

MONEY

D

MONEIS

MONEIS


Solution:

Correct Answer:

B

MONKEY

MONKEY


Question 106


Select the number from among the given options that can replace the question mark (?) in the following series.
4127, 4248, 4329, 4378, ?

दिए गए विकल्पों में से उस संख्या का चयन कीजिए, जो निम्नलिखित श्रृंखला में प्रश्नवाचक चिह्न (?) को प्रतिस्थापित कर सकती है।
4127, 4248, 4329, 4378, ?

Options

A

4403

4403

B

4344

4344

C

4430

4430

D

4304

4304


Solution:

Correct Answer:

A

4403

4403


Question 107


मोनिका ने अनु का परिचय देते हुए कहा, "वह मेरे पिता के इकलौते पुत्र की पत्नी है"। मोनिका का अनु से क्या संबंध है?

मोनिका ने अनु का परिचय देते हुए कहा, "वह मेरे पिता के इकलौते पुत्र की पत्नी है"। मोनिका का अनु से क्या संबंध है?

Options

A

भाभी

भाभी

B

पुत्री

पुत्री

C

पत्नी

पत्नी

D

भांजी

भांजी


Solution:

Correct Answer:

A

भाभी

भाभी


Question 108


Select the option that is related to the fourth figure in the same way as the first figure is related to the second figure.

उस विकल्प का चयन कीजिए जो चौथी आकृति से उसी प्रकार संबंधित है, जिस प्रकार पहली आकृति दूसरी आकृति से संबंधित है।

Options

A

B

C

D


Solution:

Correct Answer:

D


Question 109


उस विकल्प का चयन कीजिए जो तीसरे पद से ठीक उसी प्रकार संबंधित है जिस प्रकार दूसरा पद पहले पद से संबंधित है।
ईरान : रियाल :: यूनाइटेड किंगडम : ?

उस विकल्प का चयन कीजिए जो तीसरे पद से ठीक उसी प्रकार संबंधित है जिस प्रकार दूसरा पद पहले पद से संबंधित है।
ईरान : रियाल :: यूनाइटेड किंगडम : ?

Options

A

डॉलर (Dollar)

डॉलर (Dollar)

B

टका (Taka)

टका (Taka)

C

पाउंड (Pound)

पाउंड (Pound)

D

बात (Baht)

बात (Baht)


Solution:

Correct Answer:

C

पाउंड (Pound)

पाउंड (Pound)


Question 110


In a certain code language, ‘SPIDER’ is written as ‘DERSPI’. How will ‘ASYLUM’ be written in that language?

किसी निश्चित कूट भाषा में, 'SPIDER' को 'DERSPI' लिखा जाता है। उसी भाषा में 'ASYLUM' को कैसे लिखा जाएगा?

Options

A

LUMASY

LUMASY

B

LUMSAY

LUMSAY

C

MULASY

MULASY

D

MULSAY

MULSAY


Solution:

Correct Answer:

A

LUMASY

LUMASY


Question 111


Select the option that is related to the third term in the same way as the second term is related to the first term.
BIHAR : 181892 :: ASSAM : ?

उस विकल्प का चयन कीजिए जो तीसरे पद से ठीक उसी प्रकार संबंधित है, जिस प्रकार दूसरा पद पहले पद से संबंधित है।
BIHAR : 181892 :: ASSAM : ?

Options

A

14119191

14119191

B

13129191

13129191

C

13119191

13119191

D

13119291

13119291


Solution:

Correct Answer:

C

13119191

13119191


Question 112


ABCDEFGHIJKLMNOPQRSTUVWXYZ
If the 5th, 6th, 14th, 15th, 18th, 20th and 21st letters are chosen to form a meaningful word, which letter will come at the middle position? All the chosen letters must be used and each letter is to be used only once.

ABCDEFGHIJKLMNOPQRSTUVWXYZ
यदि 5वें, 6ठे, 14वें, 15वें, 18वें, 20वें और 21वें अक्षरों क एक अर्थपूर्ण शब्द बनाने के लिए चुना जाता है, तो मध्य स्थान पर कौन-सा अक्षर आएगा? सभी चयनित अक्षरों का उपयोग किया जाना चाहिए और प्रत्येक अक्षर का उपयोग केवल एक ही बार किया जाना है।

Options

A

T

T

B

R

R

C

U

U

D

E

E


Solution:

Correct Answer:

A

T

T


Question 113


Select the correct combination of mathematical signs that can sequentially replace the * signs and balance the following equation.
117 * 13 * 65 * 15 * 984

गणितीय चिह्नों के उस सही संयोजन का चयन कीजिए जो * चिह्नों को क्रमिक रूप से प्रतिस्थापित कर सकता है और निम्नलिखित समीकरण को संतुलित कर सकता है।
117 * 13 * 65 * 15 * 984

Options

A

÷, +, ×, =

÷, +, ×, =

B

+, ÷, ×, =

+, ÷, ×, =

C

×, +, ÷, =

×, +, ÷, =

D

−, ÷, +, =

−, ÷, +, =


Solution:

Correct Answer:

A

÷, +, ×, =

÷, +, ×, =


Question 114


Which of the following equations is correct?

निम्नलिखित में से कौन-सा समीकरण सही है?

Options

A

55 ÷ 11 + 6 × 6 − 8 = 43

55 ÷ 11 + 6 × 6 − 8 = 43

B

56 ÷ 7 + 3 × 4 − 7 = 13

56 ÷ 7 + 3 × 4 − 7 = 13

C

46 ÷ 23 + 3 × 2 − 4 = 14

46 ÷ 23 + 3 × 2 − 4 = 14

D

76 ÷ 19 + 9 × 8 − 9 = 65

76 ÷ 19 + 9 × 8 − 9 = 65


Solution:

Correct Answer:

B

56 ÷ 7 + 3 × 4 − 7 = 13

56 ÷ 7 + 3 × 4 − 7 = 13


Question 115


Select the number from among the given options that can replace the question mark (?) in the following series.
1464, 1631, 1798, 1965, ?

दिए गए विकल्पों में से उस संख्या का चयन कीजिए, जो निम्नलिखित श्रृंखला में प्रश्नवाचक चिह्न (?) को प्रतिस्थापित कर सकती है।
1464, 1631, 1798, 1965, ?

Options

A

2123

2123

B

2132

2132

C

2321

2321

D

2231

2231


Solution:

Correct Answer:

B

2132

2132


Question 116


In a certain code language,
‘A + B’ means ‘A is the daughter of B’,
‘A × B’ means ‘A is the father of B’,
‘A ÷ B’ means ‘A is the brother of B’.
If ‘M ÷ N + P × Q’, how is M related to Q?

एक निश्चित कूट भाषा में,
'A + B' का अर्थ है कि 'A, B की पुत्री है',
'A × B' का अर्थ है कि 'A, B के पिता हैं',
'A ÷ B' का अर्थ है कि 'A, B का भाई है'।
यदि ‘M ÷ N + P × Q’ है, तो M का Q से क्या संबंध है?

Options

A

Father’s brother

Father’s brother

B

Son

Son

C

Father

Father

D

Brother

Brother


Solution:

Correct Answer:

D

Brother

भाई


Question 117


In a certain code language, ‘RAIN’ is coded as ‘44’, and ‘CLOUD’ is coded as ‘58’. How will ‘UMBRELLA’ be coded in that language?

किसी निश्चित कूट भाषा में, ‘RAIN’ को ‘44’ के रूप में कूटबद्ध किया जाता है और ‘CLOUD’ को ‘58’ के रूप में कूटबद्ध किया जाता है। उसी भाषा में ‘UMBRELLA’ को कैसे कूटबद्ध किया जाएगा?

Options

A

84

84

B

81

81

C

98

98

D

89

89


Solution:

Correct Answer:

D

89

89


Question 118


In a certain code language,
‘A # B’ means ‘A is the son of B’,
‘A % B’ means ‘A is the daughter of B’,
‘A & B’ means ‘A is the father of B’,
‘A @ B’ means ‘A is the brother of B’.
If ‘M # N % P & Q @ T’, how is M related to T?

एक निश्चित कूट भाषा में,
'A # B' का अर्थ है कि’ A, B का पुत्र है',
'A % B' का अर्थ है कि 'A, B की पुत्री है',
'A & B' का अर्थ है कि 'A, B के पिता हैं',
'A @ B' का अर्थ है कि’ A, B का भाई है'।
यदि ‘M # N % P & Q @ T’ है, तो M का T से क्या संबंध है?

Options

A

Son

Son

B

Brother

Brother

C

Sister’s son

Sister’s son

D

Husband

Husband


Solution:

Correct Answer:

C

Sister’s son

भांजा


Question 119


Select the alphanumeric-cluster from among the given options that can replace the question mark (?) in the following series.
87K, 96P, 121U, 170Z, ?

दिए गए विकल्पों में से उस अक्षरांकीय-समूह का चयन कीजिए, जो निम्नलिखित श्रृंखला में प्रश्न चिह्न (?) को प्रतिस्थापित कर सके।
87K, 96P, 121U, 170Z, ?

Options

A

231F

231F

B

215F

215F

C

251E

251E

D

241F

241F


Solution:

Correct Answer:

C

251E

251E


Question 120


Four number-pairs have been given, out of which three are alike in some manner and one is different. Select the number-pair that is different.

चार संख्या-युग्म दिए गए हैं, जिनमें से तीन किसी न किसी तरीके से एक समान हैं और एक असंगत है। उस असंगत संख्या-युग्म का चयन कीजिए।

Options

A

17-289

17-289

B

19-381

19-381

C

11-121

11-121

D

13-169

13-169


Solution:

Correct Answer:

B

19-381

19-381


Question 121


निम्नलिखित वाक्य के अशुद्ध भाग का चयन कीजिये-
‘कर्णिका ने देखा कि बूढ़ी काकी जूठी पत्तलों को खाने लगी|’

निम्नलिखित वाक्य के अशुद्ध भाग का चयन कीजिये-
‘कर्णिका ने देखा कि बूढ़ी काकी जूठी पत्तलों को खाने लगी|’

Options

A

कर्णिका ने देखा

कर्णिका ने देखा

B

जूठी पत्तलों को

जूठी पत्तलों को

C

कि बूढ़ी काकी

कि बूढ़ी काकी

D

खाने लगी

खाने लगी


Solution:

Correct Answer:

D

खाने लगी

खाने लगी


Question 122


निम्नलिखित वाक्यों में से शुद्ध वाक्य का चयन कीजिये-

निम्नलिखित वाक्यों में से शुद्ध वाक्य का चयन कीजिये-

Options

A

अगले चौराहे के पास एक सब्जी की दूकान है|

अगले चौराहे के पास एक सब्जी की दूकान है|

B

अगले चौराहे के पास सब्जी की एक दूकान है|

अगले चौराहे के पास सब्जी की एक दूकान है|

C

अगले चौराहे में सब्जी की एक दूकान है|

अगले चौराहे में सब्जी की एक दूकान है|

D

अगले चौराहे पर एक सब्जियों की दूकान है|

अगले चौराहे पर एक सब्जियों की दूकान है|


Solution:

Correct Answer:

B

अगले चौराहे के पास सब्जी की एक दूकान है|

अगले चौराहे के पास सब्जी की एक दूकान है|


Question 123


‘नंदना’ शब्द के पर्यायवाची का चयन करें-

‘नंदना’ शब्द के पर्यायवाची का चयन करें-

Options

A

पुत्री

पुत्री

B

वधू

वधू

C

नदी

नदी

D

स्त्री

स्त्री


Solution:

Correct Answer:

A

पुत्री

पुत्री


Question 124


निम्नलिखित में से कौन-सा शब्द ‘ऐहिक ’ का विपरीतार्थक है?

निम्नलिखित में से कौन-सा शब्द ‘ऐहिक ’ का विपरीतार्थक है?

Options

A

पारलौकिक

पारलौकिक

B

अनैहिक्य

अनैहिक्य

C

अनैक्य

अनैक्य

D

ऐक्य

ऐक्य


Solution:

Correct Answer:

A

पारलौकिक

पारलौकिक


Question 125


सटीक मुहावरा चुनकर, निम्नलिखित वाक्य की पूर्ति कीजिये-
‘मुझे _______ नहीं आता, जो भी काम करता हूँ,पुख्ता करता हूँ|’

सटीक मुहावरा चुनकर, निम्नलिखित वाक्य की पूर्ति कीजिये-
‘मुझे _______ नहीं आता, जो भी काम करता हूँ,पुख्ता करता हूँ|’

Options

A

पानी की लहरें गिनना

पानी की लहरें गिनना

B

पानी में आग लगाना

पानी में आग लगाना

C

पानी छाँटना

पानी छाँटना

D

पानी पर नींव डालना

पानी पर नींव डालना


Solution:

Correct Answer:

D

पानी पर नींव डालना

पानी पर नींव डालना


Question 126


निम्नलिखित में से किस मुहावरे का अर्थ ‘कम उम्र में ही चतुर होना’ है?

निम्नलिखित में से किस मुहावरे का अर्थ ‘कम उम्र में ही चतुर होना’ है?

Options

A

पेट में दाढ़ी होना

पेट में दाढ़ी होना

B

नौ दो ग्यारह होना

नौ दो ग्यारह होना

C

लोहे के चने चबाना

लोहे के चने चबाना

D

आँखें चार होना

आँखें चार होना


Solution:

Correct Answer:

A

पेट में दाढ़ी होना

पेट में दाढ़ी होना


Question 127


निम्नलिखित में से किस मुहावरे का अर्थ ‘असंभव कार्य करना’ है?

निम्नलिखित में से किस मुहावरे का अर्थ ‘असंभव कार्य करना’ है?

Options

A

निन्यानबे के फेर में पड़ना

निन्यानबे के फेर में पड़ना

B

पापड बेलना

पापड बेलना

C

बालू से तेल निकालना

बालू से तेल निकालना

D

मन के लड्डू खाना

मन के लड्डू खाना


Solution:

Correct Answer:

C

बालू से तेल निकालना

बालू से तेल निकालना


Question 128


‘आज कार्यालय पहुँचते ही अधिकारी महोदय मुझ पर बरस गए|’
उपर्युक्त वाक्य में कौन सा भाग उचित रूप में नहीं लिखा गया है? चिह्नित कीजिये-

‘आज कार्यालय पहुँचते ही अधिकारी महोदय मुझ पर बरस गए|’
उपर्युक्त वाक्य में कौन सा भाग उचित रूप में नहीं लिखा गया है? चिह्नित कीजिये-

Options

A

आज कार्यालय

आज कार्यालय

B

अधिकारी महोदय

अधिकारी महोदय

C

मुझपर बरस गए

मुझपर बरस गए

D

पहुँचते ही

पहुँचते ही


Solution:

Correct Answer:

C

मुझपर बरस गए

मुझपर बरस गए


Question 129


निम्नलिखित में से ‘आगे कुआँ, पीछे खाईं’ कहावत की समानार्थी लोकोक्ति का चयन करें-

निम्नलिखित में से ‘आगे कुआँ, पीछे खाईं’ कहावत की समानार्थी लोकोक्ति का चयन करें-

Options

A

पहले आत्मा, फिर परमात्मा

पहले आत्मा, फिर परमात्मा

B

भई गति साँप छछूंदर केरी

भई गति साँप छछूंदर केरी

C

नाच न जाने आँगन टेढ़ा

नाच न जाने आँगन टेढ़ा

D

बिल्ली के भाग से छींका टूटा

बिल्ली के भाग से छींका टूटा


Solution:

Correct Answer:

B

भई गति साँप छछूंदर केरी

भई गति साँप छछूंदर केरी


Question 130


उपयुक्त लोकोक्ति चुनकर, वाक्य में रिक्त स्थान की पूर्ति कीजिये-
मैंने कहा थाकि गाय को चारा डाल दो और वह पानी दे आया, ये तो वही बात हुई कि _______

उपयुक्त लोकोक्ति चुनकर, वाक्य में रिक्त स्थान की पूर्ति कीजिये-
मैंने कहा थाकि गाय को चारा डाल दो और वह पानी दे आया, ये तो वही बात हुई कि _______

Options

A

कासा भर खाना, आसा भर सोना

कासा भर खाना, आसा भर सोना

B

आप मरे जग परलै

आप मरे जग परलै

C

कहें खेत की, सुनें खलिहान की

कहें खेत की, सुनें खलिहान की

D

कहीं की ईंट, कहीं का रोड़ा

कहीं की ईंट, कहीं का रोड़ा


Solution:

Correct Answer:

C

कहें खेत की, सुनें खलिहान की

कहें खेत की, सुनें खलिहान की


Question 131


‘आपकी रचना तो उत्कृष्ट कोटि की है, परन्तु इसमें वर्तनीगत अशुद्धियों की बाहुल्यता है|’
उपर्युक्त वाक्य के किस भाग में अशुद्धि है?

‘आपकी रचना तो उत्कृष्ट कोटि की है, परन्तु इसमें वर्तनीगत अशुद्धियों की बाहुल्यता है|’
उपर्युक्त वाक्य के किस भाग में अशुद्धि है?

Options

A

परन्तु इसमें वर्तनीगत

परन्तु इसमें वर्तनीगत

B

आपकी रचना तो

आपकी रचना तो

C

अशुद्धियों की बाहुल्यता है

अशुद्धियों की बाहुल्यता है

D

उत्कृष्ट कोटि की है

उत्कृष्ट कोटि की है


Solution:

Correct Answer:

C

अशुद्धियों की बाहुल्यता है

अशुद्धियों की बाहुल्यता है


Question 132


‘कंगाली में आटा गीला’– इस कहावत के उचित अर्थ का चयन करें-

‘कंगाली में आटा गीला’– इस कहावत के उचित अर्थ का चयन करें-

Options

A

नुकसान की भरपाई न होने पर कहते हैं

नुकसान की भरपाई न होने पर कहते हैं

B

बुरे दिन आने पर सब काम बिगड़ते चले जाते हैं

बुरे दिन आने पर सब काम बिगड़ते चले जाते हैं

C

गरीब का मज़ाक बनता है

गरीब का मज़ाक बनता है

D

उन्नति में बाधा पहुँचना

उन्नति में बाधा पहुँचना


Solution:

Correct Answer:

B

बुरे दिन आने पर सब काम बिगड़ते चले जाते हैं

बुरे दिन आने पर सब काम बिगड़ते चले जाते हैं


Question 133


निम्नलिखित में से कौन सा शब्द ‘बुद्ध’ का पर्यायवाची नहीं है?

निम्नलिखित में से कौन सा शब्द ‘बुद्ध’ का पर्यायवाची नहीं है?

Options

A

जागृत

जागृत

B

सिद्धार्थ

सिद्धार्थ

C

दीप्राक्ष

दीप्राक्ष

D

अमिताभ

अमिताभ


Solution:

Correct Answer:

C

दीप्राक्ष

दीप्राक्ष


Question 134


‘व्यापक’ शब्द के विलोमार्थी का चयन करें-

‘व्यापक’ शब्द के विलोमार्थी का चयन करें-

Options

A

शांत

शांत

B

अव्यर्थ

अव्यर्थ

C

संकुचित

संकुचित

D

अशक्त

अशक्त


Solution:

Correct Answer:

C

संकुचित

संकुचित


Question 135


सटीक कहावत चुनकर, निम्नलिखित कथन की पूर्ति कीजिये-
‘अपनी हर परेशानी में मुझसे मदद मांगने वाले पंकज की जब से ऊंचे पद पर नियुक्ति हुई है, सीधे मुँह बात तक नहीं करता I किसी ने ऐसे ही लोगों के लिए कहा है कि_______

सटीक कहावत चुनकर, निम्नलिखित कथन की पूर्ति कीजिये-
‘अपनी हर परेशानी में मुझसे मदद मांगने वाले पंकज की जब से ऊंचे पद पर नियुक्ति हुई है, सीधे मुँह बात तक नहीं करता I किसी ने ऐसे ही लोगों के लिए कहा है कि_______

Options

A

प्यादा से फ़रज़ी भयो, टेढ़ो- टेढ़ो जाय

प्यादा से फ़रज़ी भयो, टेढ़ो- टेढ़ो जाय

B

नेकी कर दरिया में डाल

नेकी कर दरिया में डाल

C

पर उपदेश कुशल बहुतेरे

पर उपदेश कुशल बहुतेरे

D

बन्दर के हाथ में आईना

बन्दर के हाथ में आईना


Solution:

Correct Answer:

A

प्यादा से फ़रज़ी भयो, टेढ़ो- टेढ़ो जाय

प्यादा से फ़रज़ी भयो, टेढ़ो- टेढ़ो जाय


Question 136


‘नकेल हाथ में होना’ मुहावरे के उचित अर्थ का चयन कीजिये-

‘नकेल हाथ में होना’ मुहावरे के उचित अर्थ का चयन कीजिये-

Options

A

बस में होना

बस में होना

B

स्वभाव पहचानना

स्वभाव पहचानना

C

पहुँच में होना

पहुँच में होना

D

संकट में होना

संकट में होना


Solution:

Correct Answer:

A

बस में होना

बस में होना


Question 137


‘नाट्यशाला में आज ‘काठ का घोड़ा’ नामक नाटक का खेल हुआ|’
उपर्युक्त वाक्य के किस अंश में त्रुटि है? पहचान करें-

‘नाट्यशाला में आज ‘काठ का घोड़ा’ नामक नाटक का खेल हुआ|’
उपर्युक्त वाक्य के किस अंश में त्रुटि है? पहचान करें-

Options

A

आज ‘काठ का घोड़ा’

आज ‘काठ का घोड़ा’

B

नाट्यशाला में

नाट्यशाला में

C

नामक नाटक

नामक नाटक

D

का खेल हुआ

का खेल हुआ


Solution:

Correct Answer:

D

का खेल हुआ

का खेल हुआ


Question 138


निम्नलिखित में से किस वाक्य में ‘कर्ता और क्रिया’ का अन्वय शुद्ध है?

निम्नलिखित में से किस वाक्य में ‘कर्ता और क्रिया’ का अन्वय शुद्ध है?

Options

A

सैनिक ने युद्ध में साहस और धैर्य दिखाए|

सैनिक ने युद्ध में साहस और धैर्य दिखाए|

B

गुंजन अथवा राधिका हिंदी प्रतियोगिता में भाग लेंगे|

गुंजन अथवा राधिका हिंदी प्रतियोगिता में भाग लेंगे|

C

कैकेयी और भरत वन में नहीं गए |

कैकेयी और भरत वन में नहीं गए |

D

गुरूजी आप कहाँ जा रहे हो?

गुरूजी आप कहाँ जा रहे हो?


Solution:

Correct Answer:

C

कैकेयी और भरत वन में नहीं गए |

कैकेयी और भरत वन में नहीं गए |


Question 139


निम्नलिखित शब्दों में से ‘तृषा’ शब्द के विलोमार्थी का चयन करें-

निम्नलिखित शब्दों में से ‘तृषा’ शब्द के विलोमार्थी का चयन करें-

Options

A

तिक्त

तिक्त

B

तृप्ति

तृप्ति

C

क्षुधा

क्षुधा

D

तीव्र

तीव्र


Solution:

Correct Answer:

B

तृप्ति

तृप्ति


Question 140


‘काठ की हांडी बार-बार नहीं चढ़ती’ – इस लोकोक्ति के उचित अर्थ का चयन करें-

‘काठ की हांडी बार-बार नहीं चढ़ती’ – इस लोकोक्ति के उचित अर्थ का चयन करें-

Options

A

कच्चे बर्तन में खाना एक बार ही बन सकता है

कच्चे बर्तन में खाना एक बार ही बन सकता है

B

परिश्रम बहुत अधिक, उपलब्धि अत्यंत कम

परिश्रम बहुत अधिक, उपलब्धि अत्यंत कम

C

सस्ती चीज़ बहुत दिनों तक नहीं चलती

सस्ती चीज़ बहुत दिनों तक नहीं चलती

D

धोखा एक बार तो चल जाता है, बार-बार नहीं दिया जा सकता

धोखा एक बार तो चल जाता है, बार-बार नहीं दिया जा सकता


Solution:

Correct Answer:

D

धोखा एक बार तो चल जाता है, बार-बार नहीं दिया जा सकता

धोखा एक बार तो चल जाता है, बार-बार नहीं दिया जा सकता


Question 141


निम्नलिखित में से किस वाक्य में व्याकरण सम्बन्धी अशुद्धि नहीं है?

निम्नलिखित में से किस वाक्य में व्याकरण सम्बन्धी अशुद्धि नहीं है?

Options

A

जब घर से चलें तो ताला अच्छी तरह देख लीजिए|

जब घर से चलें तो ताला अच्छी तरह देख लीजिए|

B

अगले हफ्ते विद्यालय पुनः बंद होने की संभावना है|

अगले हफ्ते विद्यालय पुनः बंद होने की संभावना है|

C

हल्का सा हवा का झोंका आया और मन उल्लसित कर गया|

हल्का सा हवा का झोंका आया और मन उल्लसित कर गया|

D

इतनी ऊँचाई पर खड़े होकर मेरा तो प्राण ही सूख गया|

इतनी ऊँचाई पर खड़े होकर मेरा तो प्राण ही सूख गया|


Solution:

Correct Answer:

B

अगले हफ्ते विद्यालय पुनः बंद होने की संभावना है|

अगले हफ्ते विद्यालय पुनः बंद होने की संभावना है|


Question 142


निम्नलिखित में से किस वाक्य में ‘कर्म और क्रिया’ का अन्वय शुद्ध है?

निम्नलिखित में से किस वाक्य में ‘कर्म और क्रिया’ का अन्वय शुद्ध है?

Options

A

दानिश को रसोई बनाना नहीं आती|

दानिश को रसोई बनाना नहीं आती|

B

तुमने आज फल, दाल, चावल और रोटियाँ खाये|

तुमने आज फल, दाल, चावल और रोटियाँ खाये|

C

रमा, विभा और अतुल शिमला गए, वह कल आ जाएँगे|

रमा, विभा और अतुल शिमला गए, वह कल आ जाएँगे|

D

हमने लेख और कहानियाँ पढ़ीं|

हमने लेख और कहानियाँ पढ़ीं|


Solution:

Correct Answer:

D

हमने लेख और कहानियाँ पढ़ीं|

हमने लेख और कहानियाँ पढ़ीं|


Question 143


निम्नलिखित में से तद्भव शब्द की पहचान कीजिए -

निम्नलिखित में से तद्भव शब्द की पहचान कीजिए -

Options

A

माता

माता

B

पाश

पाश

C

वृश्चिक

वृश्चिक

D

मक्खन

मक्खन


Solution:

Correct Answer:

D

मक्खन

मक्खन


Question 144


निम्नलिखित में से किस विकल्प के सभी शब्द ‘बहेलिया’ के पर्याय हैं?

निम्नलिखित में से किस विकल्प के सभी शब्द ‘बहेलिया’ के पर्याय हैं?

Options

A

बहली, शामित्र, प्रवेणी

बहली, शामित्र, प्रवेणी

B

लुब्धक, व्याध, अहेरी

लुब्धक, व्याध, अहेरी

C

कुहर, चिड़ीमार, नीरज

कुहर, चिड़ीमार, नीरज

D

आखेटक, अलक, हलायुध

आखेटक, अलक, हलायुध


Solution:

Correct Answer:

B

लुब्धक, व्याध, अहेरी

लुब्धक, व्याध, अहेरी


Question 145


निम्नलिखित में से कौन सा विकल्प शुद्ध तद्भव- तत्सम युग्म है?

निम्नलिखित में से कौन सा विकल्प शुद्ध तद्भव- तत्सम युग्म है?

Options

A

पिपासा – प्यास

पिपासा – प्यास

B

बुआ – पित्रश्वसा

बुआ – पित्रश्वसा

C

तुंद – तोंद

तुंद – तोंद

D

पिटारा – पर्पट

पिटारा – पर्पट


Solution:

Correct Answer:

B

बुआ – पित्रश्वसा

बुआ – पित्रश्वसा


Question 146


तत्सम शब्द ‘वार्ताक’ के तद्भव रूप का चयन कीजिए -

तत्सम शब्द ‘वार्ताक’ के तद्भव रूप का चयन कीजिए -

Options

A

वातुलक

वातुलक

B

बैंगन

बैंगन

C

बौना

बौना

D

बावला

बावला


Solution:

Correct Answer:

B

बैंगन

बैंगन


Question 147


‘आनुषंगिक‘के विलोम शब्द का चयन करें-

‘आनुषंगिक‘के विलोम शब्द का चयन करें-

Options

A

अल्पजीवी

अल्पजीवी

B

प्रमुख

प्रमुख

C

निरामिष

निरामिष

D

विमुख

विमुख


Solution:

Correct Answer:

B

प्रमुख

प्रमुख


Question 148


‘यहाँ आकर भारतीय संस्कृति को मैंने गाँवों-गावों में देखा|’
उपर्युक्त वाक्य के अशुद्ध भाग का चयन कीजिये-

‘यहाँ आकर भारतीय संस्कृति को मैंने गाँवों-गावों में देखा|’
उपर्युक्त वाक्य के अशुद्ध भाग का चयन कीजिये-

Options

A

यहाँ आकर

यहाँ आकर

B

भारतीय संस्कृति को

भारतीय संस्कृति को

C

में देखा

में देखा

D

मैंने गाँवों-गावों

मैंने गाँवों-गावों


Solution:

Correct Answer:

D

मैंने गाँवों-गावों

मैंने गाँवों-गावों


Question 149


निम्नलिखित वाक्य के अशुद्ध भाग को चिह्नित कीजिए –
‘आदेश! बाहर बहुत ठंड है, तुम हाथों में दस्ताने पहन लो|’

निम्नलिखित वाक्य के अशुद्ध भाग को चिह्नित कीजिए –
‘आदेश! बाहर बहुत ठंड है, तुम हाथों में दस्ताने पहन लो|’

Options

A

तुम हाथों में दस्ताने

तुम हाथों में दस्ताने

B

आदेश!

आदेश!

C

बाहर बहुत ठंड है,

बाहर बहुत ठंड है,

D

पहन लो

पहन लो


Solution:

Correct Answer:

A

तुम हाथों में दस्ताने

तुम हाथों में दस्ताने


Question 150


‘जीभ के नीचे जीभ होना’ मुहावरे के उचित अर्थ का चयन करें-

‘जीभ के नीचे जीभ होना’ मुहावरे के उचित अर्थ का चयन करें-

Options

A

शेखी बघारना

शेखी बघारना

B

कह कर बदल जाना

कह कर बदल जाना

C

चटक-मटक खाने की इच्छा होना

चटक-मटक खाने की इच्छा होना

D

धृष्टता से उत्तर देना

धृष्टता से उत्तर देना


Solution:

Correct Answer:

B

कह कर बदल जाना

कह कर बदल जाना


Question 151


निम्नलिखित में से ‘विष्णु’ के पर्यायवाची शब्द-समूह का चयन कीजिए-

निम्नलिखित में से ‘विष्णु’ के पर्यायवाची शब्द-समूह का चयन कीजिए-

Options

A

सुरेश, उमेश

सुरेश, उमेश

B

विश्वेश, महेश

विश्वेश, महेश

C

महेंद्र, हृषिकेश

महेंद्र, हृषिकेश

D

विरंचि, महेश्वर

विरंचि, महेश्वर


Solution:

Correct Answer:

C

महेंद्र, हृषिकेश

महेंद्र, हृषिकेश


Question 152


निम्नलिखित वाक्यों में से शुद्ध वाक्य की पहचान कीजिये-

निम्नलिखित वाक्यों में से शुद्ध वाक्य की पहचान कीजिये-

Options

A

इस कार्य में देर होना स्वाभाविक था |

इस कार्य में देर होना स्वाभाविक था |

B

इस परिसर में अनाधिकार प्रवेश वर्जित है|

इस परिसर में अनाधिकार प्रवेश वर्जित है|

C

मैंने तुम्हारे को बोला था कि जल्दी आओ|

मैंने तुम्हारे को बोला था कि जल्दी आओ|

D

इस राह पर पग-पग में काँटे मिलेंगे|

इस राह पर पग-पग में काँटे मिलेंगे|


Solution:

Correct Answer:

A

इस कार्य में देर होना स्वाभाविक था |

इस कार्य में देर होना स्वाभाविक था |


Question 153


निम्नलिखित वाक्यों में से अशुद्ध वाक्य का चयन कीजिये-

निम्नलिखित वाक्यों में से अशुद्ध वाक्य का चयन कीजिये-

Options

A

मुझे अचार अच्छा लगता है|

मुझे अचार अच्छा लगता है|

B

तुम्हें यहाँ नहीं आना चाहिए|

तुम्हें यहाँ नहीं आना चाहिए|

C

सभी बच्चा इस समय खेलना चाहते हैं|

सभी बच्चा इस समय खेलना चाहते हैं|

D

उसे वहाँ अपनी पसंद का काम मिल गया|

उसे वहाँ अपनी पसंद का काम मिल गया|


Solution:

Correct Answer:

C

सभी बच्चा इस समय खेलना चाहते हैं|

सभी बच्चा इस समय खेलना चाहते हैं|


Question 154


निम्नलिखित वाक्य के अशुद्ध भाग को चिह्नित कीजिए –
‘हमने तो पहले ही तुम्हें मना करा था कि उस बढ़ई का काम अच्छा नहीं है|’

निम्नलिखित वाक्य के अशुद्ध भाग को चिह्नित कीजिए –
‘हमने तो पहले ही तुम्हें मना करा था कि उस बढ़ई का काम अच्छा नहीं है|’

Options

A

कि उस बढ़ई का काम

कि उस बढ़ई का काम

B

हमने तो पहले ही

हमने तो पहले ही

C

अच्छा नहीं है

अच्छा नहीं है

D

तुम्हें मना करा था

तुम्हें मना करा था


Solution:

Correct Answer:

D

तुम्हें मना करा था

तुम्हें मना करा था


Question 155


निम्नलिखित वाक्यों में से शुद्ध वाक्य का चयन कीजिये-

निम्नलिखित वाक्यों में से शुद्ध वाक्य का चयन कीजिये-

Options

A

यह मेरी सहेली है यह मेरे साथ खेलती है|

यह मेरी सहेली है यह मेरे साथ खेलती है|

B

तुम तुम्हारे घर जा सकते हो|

तुम तुम्हारे घर जा सकते हो|

C

यह उनकी समझ में नहीं आएगा |

यह उनकी समझ में नहीं आएगा |

D

उसने यही कहना था कि तुम कुछ न करो |

उसने यही कहना था कि तुम कुछ न करो |


Solution:

Correct Answer:

C

यह उनकी समझ में नहीं आएगा |

यह उनकी समझ में नहीं आएगा |


Question 156


निम्नलिखित में से कौन सा शब्द ‘अक्षर’ का पर्यायवाची नहीं है?

निम्नलिखित में से कौन सा शब्द ‘अक्षर’ का पर्यायवाची नहीं है?

Options

A

अमर

अमर

B

ब्रह्म

ब्रह्म

C

समस्त

समस्त

D

वर्ण

वर्ण


Solution:

Correct Answer:

C

समस्त

समस्त


Question 157


निम्नलिखित में से अशुद्ध वर्तनी वाले तत्सम शब्द का चयन करें-

निम्नलिखित में से अशुद्ध वर्तनी वाले तत्सम शब्द का चयन करें-

Options

A

द्राक्षा

द्राक्षा

B

नयन

नयन

C

दर्सन

दर्सन

D

दशम

दशम


Solution:

Correct Answer:

C

दर्सन

दर्सन


Question 158


निम्नलिखित में से अशुद्ध विलोम- युग्म की पहचान करें-

निम्नलिखित में से अशुद्ध विलोम- युग्म की पहचान करें-

Options

A

अनुराग – विराग

अनुराग – विराग

B

कृतज्ञ – कृतघ्न

कृतज्ञ – कृतघ्न

C

अभिराम – अविराम

अभिराम – अविराम

D

अभिज्ञ – अनभिज्ञ

अभिज्ञ – अनभिज्ञ


Solution:

Correct Answer:

C

अभिराम – अविराम

अभिराम – अविराम


Question 159


निम्नलिखित में से किस विकल्प में लोकोक्ति का प्रयोग अशुद्ध है?

निम्नलिखित में से किस विकल्प में लोकोक्ति का प्रयोग अशुद्ध है?

Options

A

प्रशासन का वरदहस्त होने से जमना प्रसाद राज्य भर के पुलिस-तंत्र से भिड़कर ‘हिमायती की घोड़ी, ऐराकी को लात मारे’ वाली कहावत चरितार्थ कर रहा है|

प्रशासन का वरदहस्त होने से जमना प्रसाद राज्य भर के पुलिस-तंत्र से भिड़कर ‘हिमायती की घोड़ी, ऐराकी को लात मारे’ वाली कहावत चरितार्थ कर रहा है|

B

अभी तो मैंने कोई जमीन खरीदी भी नहीं और तुम दोनों उसपर अपने हिस्से के लिए लड़ रहे हो! इसे कहते हैं कि ‘सूत न कपास, जुलाहों में लट्ठम-लट्ठा |’

अभी तो मैंने कोई जमीन खरीदी भी नहीं और तुम दोनों उसपर अपने हिस्से के लिए लड़ रहे हो! इसे कहते हैं कि ‘सूत न कपास, जुलाहों में लट्ठम-लट्ठा |’

C

जीवन में लाख कष्ट आने पर भी घबराना नहीं चाहिए, क्योंकि ‘हथेली पर सरसों नहीं जमती|’

जीवन में लाख कष्ट आने पर भी घबराना नहीं चाहिए, क्योंकि ‘हथेली पर सरसों नहीं जमती|’

D

सच्चे और ईमानदार व्यक्ति की कितनी भी बुराई करने का प्रयास किया जाए, उसपर कोई आंच नहीं आती, क्योंकि ‘सूरज धूल डालने से नहीं छिपता|’

सच्चे और ईमानदार व्यक्ति की कितनी भी बुराई करने का प्रयास किया जाए, उसपर कोई आंच नहीं आती, क्योंकि ‘सूरज धूल डालने से नहीं छिपता|’


Solution:

Correct Answer:

C

जीवन में लाख कष्ट आने पर भी घबराना नहीं चाहिए, क्योंकि ‘हथेली पर सरसों नहीं जमती|’

जीवन में लाख कष्ट आने पर भी घबराना नहीं चाहिए, क्योंकि ‘हथेली पर सरसों नहीं जमती|’


Question 160


‘अंगुष्ठिका’ शब्द के शुद्ध तद्भव रूप का चयन कीजिए -

‘अंगुष्ठिका’ शब्द के शुद्ध तद्भव रूप का चयन कीजिए -

Options

A

अँगूष्ठी

अँगूष्ठी

B

अंगूठि

अंगूठि

C

अंगुठी

अंगुठी

D

अँगूठी

अँगूठी


Solution:

Correct Answer:

D

अँगूठी

अँगूठी


Question 161


‘कै हंसा मोती चुगै, कै भूखा मर जाय’ कहावत निम्नलिखित में से किस कथन के लिए उपयुक्त है?

‘कै हंसा मोती चुगै, कै भूखा मर जाय’ कहावत निम्नलिखित में से किस कथन के लिए उपयुक्त है?

Options

A

कई बार असफल होने पर भी रमा ने अपना विषय नहीं बदला, क्योंकि उसका कहना है कि या तो वह इसी विषय में स्नातक करेगी या करेगी ही नहीं|

कई बार असफल होने पर भी रमा ने अपना विषय नहीं बदला, क्योंकि उसका कहना है कि या तो वह इसी विषय में स्नातक करेगी या करेगी ही नहीं|

B

व्यक्ति का स्वभाव कभी नहीं बदलता, चाहे वह मर ही क्यों न जाय|

व्यक्ति का स्वभाव कभी नहीं बदलता, चाहे वह मर ही क्यों न जाय|

C

जो कुछ अपने अधिकार में नहीं है, उसे पाने के लिए ही अड़े रहना कहाँ की बुद्धिमानी है !

जो कुछ अपने अधिकार में नहीं है, उसे पाने के लिए ही अड़े रहना कहाँ की बुद्धिमानी है !

D

जो व्यक्ति घमंड करता है उसे कभी न कभी झुकना ही पड़ता है |

जो व्यक्ति घमंड करता है उसे कभी न कभी झुकना ही पड़ता है |


Solution:

Correct Answer:

A

कई बार असफल होने पर भी रमा ने अपना विषय नहीं बदला, क्योंकि उसका कहना है कि या तो वह इसी विषय में स्नातक करेगी या करेगी ही नहीं|

कई बार असफल होने पर भी रमा ने अपना विषय नहीं बदला, क्योंकि उसका कहना है कि या तो वह इसी विषय में स्नातक करेगी या करेगी ही नहीं|


Question 162


निम्नलिखित में से किस वाक्य में कोई अशुद्धि नहीं है?

निम्नलिखित में से किस वाक्य में कोई अशुद्धि नहीं है?

Options

A

यौवन की बुराइयों से बचो|

यौवन की बुराइयों से बचो|

B

अधिकांश लोगों का यही हाल है|

अधिकांश लोगों का यही हाल है|

C

यहाँ उपस्थित सभी लोगों में सविता अत्यंत श्रेष्ठ है|

यहाँ उपस्थित सभी लोगों में सविता अत्यंत श्रेष्ठ है|

D

मध्यकालीन युग में वास्तुकलाओं की अत्यंत उन्नति हुई|

मध्यकालीन युग में वास्तुकलाओं की अत्यंत उन्नति हुई|


Solution:

Correct Answer:

A

यौवन की बुराइयों से बचो|

यौवन की बुराइयों से बचो|


Question 163


‘खटराग में पड़ना’ मुहावरे के अर्थ का चयन कीजिये-

‘खटराग में पड़ना’ मुहावरे के अर्थ का चयन कीजिये-

Options

A

दुविधा में फँसना

दुविधा में फँसना

B

आडम्बर करना

आडम्बर करना

C

बखेड़े में पड़ना

बखेड़े में पड़ना

D

मक्कारी करना

मक्कारी करना


Solution:

Correct Answer:

C

बखेड़े में पड़ना

बखेड़े में पड़ना


Question 164


‘कौमुदी’ शब्द के पर्यायवाची का चयन कीजिए -

‘कौमुदी’ शब्द के पर्यायवाची का चयन कीजिए -

Options

A

रोशनी

रोशनी

B

चाँदी

चाँदी

C

चाँद

चाँद

D

ज्योत्स्ना

ज्योत्स्ना


Solution:

Correct Answer:

D

ज्योत्स्ना

ज्योत्स्ना


Question 165


‘कल कौन सी चीज़ से चोट लगी थी कि तुम्हें तुरंत अस्पताल ले जाना पड़ा|’
उपर्युक्त वाक्य के किस भाग में अशुद्धि है?

‘कल कौन सी चीज़ से चोट लगी थी कि तुम्हें तुरंत अस्पताल ले जाना पड़ा|’
उपर्युक्त वाक्य के किस भाग में अशुद्धि है?

Options

A

कि तुम्हें तुरंत अस्पताल

कि तुम्हें तुरंत अस्पताल

B

कल कौन सी चीज़ से

कल कौन सी चीज़ से

C

ले जाना पड़ा|

ले जाना पड़ा|

D

चोट लगी थी

चोट लगी थी


Solution:

Correct Answer:

B

कल कौन सी चीज़ से

कल कौन सी चीज़ से


Question 166


‘घूंघट’ शब्द के उचित पर्यायवाची का चयन करें-

‘घूंघट’ शब्द के उचित पर्यायवाची का चयन करें-

Options

A

किंकिर

किंकिर

B

अब्र

अब्र

C

अलल

अलल

D

अवगुंठन

अवगुंठन


Solution:

Correct Answer:

D

अवगुंठन

अवगुंठन


Question 167


निम्नलिखित में से ‘आह्लाद’ के विलोमार्थी शब्द का चयन करें-

निम्नलिखित में से ‘आह्लाद’ के विलोमार्थी शब्द का चयन करें-

Options

A

आहूत

आहूत

B

विषाद

विषाद

C

प्रहलाद

प्रहलाद

D

निषाद

निषाद


Solution:

Correct Answer:

B

विषाद

विषाद


Question 168


निम्नलिखित में से किस विकल्प के दोनों शब्द आपस में पर्यायवाची हैं?

निम्नलिखित में से किस विकल्प के दोनों शब्द आपस में पर्यायवाची हैं?

Options

A

पंक, अंशु

पंक, अंशु

B

अनी, दीन

अनी, दीन

C

कृष्ण, अच्युत

कृष्ण, अच्युत

D

अनुचित, अशिष्ट

अनुचित, अशिष्ट


Solution:

Correct Answer:

C

कृष्ण, अच्युत

कृष्ण, अच्युत


Question 169


निम्नलिखित में से ‘ओस का मोती होना’ मुहावरे का उचित अर्थ क्या है?

निम्नलिखित में से ‘ओस का मोती होना’ मुहावरे का उचित अर्थ क्या है?

Options

A

सब पर समान दृष्टि रखना

सब पर समान दृष्टि रखना

B

तुच्छ या नगण्य व्यक्ति

तुच्छ या नगण्य व्यक्ति

C

शीघ्र नष्ट हो जाने वाला सौन्दर्य

शीघ्र नष्ट हो जाने वाला सौन्दर्य

D

अनुपम सौन्दर्य होना

अनुपम सौन्दर्य होना


Solution:

Correct Answer:

C

शीघ्र नष्ट हो जाने वाला सौन्दर्य

शीघ्र नष्ट हो जाने वाला सौन्दर्य


Question 170


निम्नलिखित शब्द के शुद्ध तत्सम रूप का चयन करें-
ओढ़ना

निम्नलिखित शब्द के शुद्ध तत्सम रूप का चयन करें-
ओढ़ना

Options

A

उपविष्टन

उपविष्टन

B

उपरिवसन

उपरिवसन

C

उपष्टन

उपष्टन

D

उपवेष्टन

उपवेष्टन


Solution:

Correct Answer:

D

उपवेष्टन

उपवेष्टन


Question 171


निम्नलिखित में से किस विकल्प में मुहावरे का प्रयोग अशुद्ध है?

निम्नलिखित में से किस विकल्प में मुहावरे का प्रयोग अशुद्ध है?

Options

A

मुझे जली-कटी मत सुनाओ, किसकी गलती है उसे कहो|

मुझे जली-कटी मत सुनाओ, किसकी गलती है उसे कहो|

B

थकावट से चूर होकर वह पेड़ की छाया में अंटी मार गया|

थकावट से चूर होकर वह पेड़ की छाया में अंटी मार गया|

C

वह आजकल किसी की भी बात नहीं सुनता, पैसे जेब में आते ही उसकी आँखों में चर्बी छा गयी है|

वह आजकल किसी की भी बात नहीं सुनता, पैसे जेब में आते ही उसकी आँखों में चर्बी छा गयी है|

D

अरे! वह बिलकुल अंधी खोपड़ी है, उसे कुछ भी समझाना बेकार है|

अरे! वह बिलकुल अंधी खोपड़ी है, उसे कुछ भी समझाना बेकार है|


Solution:

Correct Answer:

B

थकावट से चूर होकर वह पेड़ की छाया में अंटी मार गया|

थकावट से चूर होकर वह पेड़ की छाया में अंटी मार गया|


Question 172


‘पूत के पाँव पालने में पहचाने जाते हैं’ की समानार्थी लोकोक्ति का चयन करें-

‘पूत के पाँव पालने में पहचाने जाते हैं’ की समानार्थी लोकोक्ति का चयन करें-

Options

A

पुण्य की जड़ सदा हरी

पुण्य की जड़ सदा हरी

B

पेड़ फल से जाना जाता है

पेड़ फल से जाना जाता है

C

होनहार बिरवान के होत चीकने पात

होनहार बिरवान के होत चीकने पात

D

बोया गेहूं, उपजा जौ

बोया गेहूं, उपजा जौ


Solution:

Correct Answer:

C

होनहार बिरवान के होत चीकने पात

होनहार बिरवान के होत चीकने पात


Question 173


निम्नलिखित में से तत्सम शब्द की पहचान कीजिए -

निम्नलिखित में से तत्सम शब्द की पहचान कीजिए -

Options

A

पीपल

पीपल

B

बिंदु

बिंदु

C

नब्बे

नब्बे

D

बत्ती

बत्ती


Solution:

Correct Answer:

B

बिंदु

बिंदु


Question 174


निम्नलिखित में से कौन सा विलोम- युग्म शुद्ध है?

निम्नलिखित में से कौन सा विलोम- युग्म शुद्ध है?

Options

A

स्थापन – रोपण

स्थापन – रोपण

B

ऋत – अऋत

ऋत – अऋत

C

अधित्यका – उपत्यका

अधित्यका – उपत्यका

D

एकाग्र – समग्र

एकाग्र – समग्र


Solution:

Correct Answer:

C

अधित्यका – उपत्यका

अधित्यका – उपत्यका


Question 175


निम्नलिखित में से अशुद्ध विलोम- युग्म की पहचान करें-

निम्नलिखित में से अशुद्ध विलोम- युग्म की पहचान करें-

Options

A

कृपण – उदार

कृपण – उदार

B

ऐक्षिक – अनिवार्य

ऐक्षिक – अनिवार्य

C

प्रवृद्ध – कृश

प्रवृद्ध – कृश

D

पीन – पुष्ट

पीन – पुष्ट


Solution:

Correct Answer:

D

पीन – पुष्ट

पीन – पुष्ट


Question 176


Select the most appropriate option to fill in the blank.

I don’t believe Karuna’s story, I’m sure she ______ .

Select the most appropriate option to fill in the blank.

I don’t believe Karuna’s story, I’m sure she ______ .

Options

A

made it on

made it on

B

made it up

made it up

C

made it out

made it out

D

made it off

made it off


Solution:

Correct Answer:

B

made it up

made it up


Question 177


Parts of the following sentence have been given as options. Select the option that contains a spelling error. If you don’t find any error, mark ‘No error’ as your answer.

The repetition of events like earthquakes encourage people to become supersticious.

Parts of the following sentence have been given as options. Select the option that contains a spelling error. If you don’t find any error, mark ‘No error’ as your answer.

The repetition of events like earthquakes encourage people to become supersticious.

Options

A

The repetition of events

The repetition of events

B

to become supersticious

to become supersticious

C

No error

No error

D

like earthquakes encourage people

like earthquakes encourage people


Solution:

Correct Answer:

B

to become supersticious

to become supersticious


Question 178


Select the most appropriate option to fill in the blank.

The teacher made the children ______ their lines for the annual day function.

Select the most appropriate option to fill in the blank.

The teacher made the children ______ their lines for the annual day function.

Options

A

practise

practise

B

practised

practised

C

practising

practising

D

to practise

to practise


Solution:

Correct Answer:

A

practise

practise


Question 179


Select the most appropriate option to substitute the underlined word in the given sentence. If there is no need to substitute it, select ‘No substitution required’.

We must operate a staff meeting sometime next week.

Select the most appropriate option to substitute the underlined word in the given sentence. If there is no need to substitute it, select ‘No substitution required’.

We must operate a staff meeting sometime next week.

Options

A

establish

establish

B

hold

hold

C

assemble

assemble

D

No substitution required

No substitution required


Solution:

Correct Answer:

B

hold

hold


Question 180


Select the option which is NOT an antonym of another word by way of adding the prefix ‘Dis-’.

Select the option which is NOT an antonym of another word by way of adding the prefix ‘Dis-’.

Options

A

Disembark

Disembark

B

Disgrace

Disgrace

C

Disenchant

Disenchant

D

Disaster

Disaster


Solution:

Correct Answer:

D

Disaster

Disaster


Question 181


Select the most appropriate option to fill in the blank.

Neeraj likes to travel, ______, his wife prefers to stay at home.

Select the most appropriate option to fill in the blank.

Neeraj likes to travel, ______, his wife prefers to stay at home.

Options

A

whereas

whereas

B

otherwise

otherwise

C

in case

in case

D

moreover

moreover


Solution:

Correct Answer:

A

whereas

whereas


Question 182


Select the most appropriate option to collocate with the word ‘round of’ to fill in the blank.

Let us give Miss Chanu a big round of ______ on her feat in the Olympics.

Select the most appropriate option to collocate with the word ‘round of’ to fill in the blank.

Let us give Miss Chanu a big round of ______ on her feat in the Olympics.

Options

A

praise

praise

B

clap

clap

C

hand

hand

D

applause

applause


Solution:

Correct Answer:

D

applause

applause


Question 183


Select the most appropriate meaning of the given idiom.

Bottom line

Select the most appropriate meaning of the given idiom.

Bottom line

Options

A

A hidden agenda

A hidden agenda

B

An impossible task

An impossible task

C

An awkward situation

An awkward situation

D

The most important fact

The most important fact


Solution:

Correct Answer:

D

The most important fact

The most important fact


Question 184


Four sentences have been given below out of which three are alike in some manner and one is different. Select the one that is different.

Four sentences have been given below out of which three are alike in some manner and one is different. Select the one that is different.

Options

A

It was a ‘lofty and beautiful’ monument containing treasures made of gold, silver, emeralds, pearls, ebony and ivory.

It was a ‘lofty and beautiful’ monument containing treasures made of gold, silver, emeralds, pearls, ebony and ivory.

B

The scroll records a vast amount of hidden gold and silver treasure.

The scroll records a vast amount of hidden gold and silver treasure.

C

The writer Plutarch wrote that the tomb was located near a temple of Isis, an Egyptian Goddess.

The writer Plutarch wrote that the tomb was located near a temple of Isis, an Egyptian Goddess.

D

Ancient writers claim that Cleopatra was buried in a tomb after her death in 30 BC.

Ancient writers claim that Cleopatra was buried in a tomb after her death in 30 BC.


Solution:

Correct Answer:

B

The scroll records a vast amount of hidden gold and silver treasure.

The scroll records a vast amount of hidden gold and silver treasure.


Question 185


Select the option that expresses the given sentence in active voice.

The Chinese attack near Chushul airport in Ladakh was repulsed by Major Thapa and his men.

Select the option that expresses the given sentence in active voice.

The Chinese attack near Chushul airport in Ladakh was repulsed by Major Thapa and his men.

Options

A

The Chinese attack has repulsed Chushul airport in Ladakh near Major Thapa and his men.

The Chinese attack has repulsed Chushul airport in Ladakh near Major Thapa and his men.

B

Major Thapa and his men have repulsed the Chinese attack near Chushul airport in Ladakh.

Major Thapa and his men have repulsed the Chinese attack near Chushul airport in Ladakh.

C

Major Thapa and his men had repulsed the Chinese attack near Chushul airport in Ladakh.

Major Thapa and his men had repulsed the Chinese attack near Chushul airport in Ladakh.

D

Major Thapa and his men repulsed the Chinese attack near Chushul airport in Ladakh.

Major Thapa and his men repulsed the Chinese attack near Chushul airport in Ladakh.


Solution:

Correct Answer:

D

Major Thapa and his men repulsed the Chinese attack near Chushul airport in Ladakh.

Major Thapa and his men repulsed the Chinese attack near Chushul airport in Ladakh.


Question 186


Select the option that completes the given proverb correctly.

______ is thicker than water

Select the option that completes the given proverb correctly.

______ is thicker than water

Options

A

Oil

Oil

B

Juice

Juice

C

Milk

Milk

D

Blood

Blood


Solution:

Correct Answer:

D

Blood

Blood


Question 187


Select the most appropriate option to fill in the blank.

Go straight on, ______ you reach a round-about and then take a right turn.

Select the most appropriate option to fill in the blank.

Go straight on, ______ you reach a round-about and then take a right turn.

Options

A

since

since

B

when

when

C

from

from

D

till

till


Solution:

Correct Answer:

D

till

till


Question 188


Select the most appropriate option to fill in the blank.

He took objection on the tone ______ which she was talking to her mother.

Select the most appropriate option to fill in the blank.

He took objection on the tone ______ which she was talking to her mother.

Options

A

on

on

B

in

in

C

to

to

D

at

at


Solution:

Correct Answer:

B

in

in


Question 189


Select the option that expresses the given sentence in passive voice.

The Chinese opened a barrage of artillery and mortar fire over Sirijap post -1 in Ladakh.

Select the option that expresses the given sentence in passive voice.

The Chinese opened a barrage of artillery and mortar fire over Sirijap post -1 in Ladakh.

Options

A

A barrage of artillery and mortar fire is being opened by the Chinese over Sirijap post -1 in Ladakh.

A barrage of artillery and mortar fire is being opened by the Chinese over Sirijap post -1 in Ladakh.

B

A barrage of artillery and mortar fire was opened by the Chinese over Sirijap post -1 in Ladakh.

A barrage of artillery and mortar fire was opened by the Chinese over Sirijap post -1 in Ladakh.

C

Sirijap post -1 in Ladakh was opened by the Chinese over a barrage of artillery and mortar fire.

Sirijap post -1 in Ladakh was opened by the Chinese over a barrage of artillery and mortar fire.

D

A barrage of artillery and mortar fire has been opened by the Chinese over Sirijap post -1 in Ladakh.

A barrage of artillery and mortar fire has been opened by the Chinese over Sirijap post -1 in Ladakh.


Solution:

Correct Answer:

B

A barrage of artillery and mortar fire was opened by the Chinese over Sirijap post -1 in Ladakh.

A barrage of artillery and mortar fire was opened by the Chinese over Sirijap post -1 in Ladakh.


Question 190


Select the option that expresses the given sentence in direct speech.

I asked my guest whether he had a good sleep the previous night.

Select the option that expresses the given sentence in direct speech.

I asked my guest whether he had a good sleep the previous night.

Options

A

I said to my guest, “Did you have a good sleep last night?”

I said to my guest, “Did you have a good sleep last night?”

B

I said to my guest, “Whether he had a good sleep the previous night.”

I said to my guest, “Whether he had a good sleep the previous night.”

C

I said to my guest, “You had a good sleep last night.”

I said to my guest, “You had a good sleep last night.”

D

I said to my guest, “Did you have a good sleep the previous night?”

I said to my guest, “Did you have a good sleep the previous night?”


Solution:

Correct Answer:

A

I said to my guest, “Did you have a good sleep last night?”

I said to my guest, “Did you have a good sleep last night?”


Question 191


Four sentences of a paragraph are given below in jumbled order. Arrange the sentences in the correct order to form a meaningful and coherent paragraph.

A. The petrol bunk where I filled petrol from had a service station beside it.
B. There were two boys, perhaps fourteen years of age, who worked there.
C. One was called Ram and the other one was Gopal.
D. They were identical twins.

Four sentences of a paragraph are given below in jumbled order. Arrange the sentences in the correct order to form a meaningful and coherent paragraph.

A. The petrol bunk where I filled petrol from had a service station beside it.
B. There were two boys, perhaps fourteen years of age, who worked there.
C. One was called Ram and the other one was Gopal.
D. They were identical twins.

Options

A

ACDB

ACDB

B

ABDC

ABDC

C

ACBD

ACBD

D

ADCB

ADCB


Solution:

Correct Answer:

B

ABDC

ABDC


Question 192


Select the most appropriate option to fill in the blank.

My secretary ______ the office one hour ago. She has just come back.

Select the most appropriate option to fill in the blank.

My secretary ______ the office one hour ago. She has just come back.

Options

A

has left

has left

B

was leaving

was leaving

C

had been leaving

had been leaving

D

left

left


Solution:

Correct Answer:

D

left

left


Question 193


Select the most appropriate option to fill in the blank.

My uncle belongs to a ______ of goldsmiths in Jaipur.

Select the most appropriate option to fill in the blank.

My uncle belongs to a ______ of goldsmiths in Jaipur.

Options

A

gilled

gilled

B

gild

gild

C

guilt

guilt

D

guild

guild


Solution:

Correct Answer:

D

guild

guild


Question 194


Select the most appropriate option to fill in the blank.

When he threw a stone at me, I ducked ______ and saved my head.

Select the most appropriate option to fill in the blank.

When he threw a stone at me, I ducked ______ and saved my head.

Options

A

much

much

B

a few

a few

C

some

some

D

a little

a little


Solution:

Correct Answer:

D

a little

a little


Question 195


Select the most appropriate option to fill in the blanks.

The ______ was digging a ______ rock when he found a small yellow diamond.

Select the most appropriate option to fill in the blanks.

The ______ was digging a ______ rock when he found a small yellow diamond.

Options

A

miner; miner

miner; miner

B

minor; minor

minor; minor

C

miner; minor

miner; minor

D

minor; miner

minor; miner


Solution:

Correct Answer:

C

miner; minor

miner; minor


Question 196


Select the most appropriate option to fill in the blanks.

If I ______ the way to the ______ Metro station at Gandhi Nagar, I would go on my own.

Select the most appropriate option to fill in the blanks.

If I ______ the way to the ______ Metro station at Gandhi Nagar, I would go on my own.

Options

A

knew; new

knew; new

B

new; knew

new; knew

C

knew; knew

knew; knew

D

new; new

new; new


Solution:

Correct Answer:

A

knew; new

knew; new


Question 197


Select the most appropriate synonym of the given word.
Trivial

Select the most appropriate synonym of the given word.
Trivial

Options

A

Immaterial

Immaterial

B

Immense

Immense

C

Immobile

Immobile

D

Improper

Improper


Solution:

Correct Answer:

A

Immaterial

Immaterial


Question 198


Select the most appropriate synonym of the given word.
Rake

Select the most appropriate synonym of the given word.
Rake

Options

A

Plant

Plant

B

Throw

Throw

C

Disperse

Disperse

D

Search

Search


Solution:

Correct Answer:

D

Search

Search


Question 199


Select the most appropriate option to fill in the blank.

She felt quite ______ after her own outburst during the staff meeting.

Select the most appropriate option to fill in the blank.

She felt quite ______ after her own outburst during the staff meeting.

Options

A

embarrased

embarrased

B

embarassed

embarassed

C

emberrassed

emberrassed

D

embarrassed

embarrassed


Solution:

Correct Answer:

D

embarrassed

embarrassed


Question 200


Select the most appropriate option to fill in the blank.

I was ______ nauseated at the sight of blood and violence all around me.

Select the most appropriate option to fill in the blank.

I was ______ nauseated at the sight of blood and violence all around me.

Options

A

quietly

quietly

B

badly

badly

C

cautiously

cautiously

D

freely

freely


Solution:

Correct Answer:

B

badly

badly


Question 201


Select the most appropriate ANTONYM of the given word.
Mitigate

Select the most appropriate ANTONYM of the given word.
Mitigate

Options

A

Sharpen

Sharpen

B

Lighten

Lighten

C

Relieve

Relieve

D

Soothe

Soothe


Solution:

Correct Answer:

A

Sharpen

Sharpen


Question 202


Select the most appropriate option to fill in the blanks.

Don’t ______ any dress material. Her dress is tight at the _______.

Select the most appropriate option to fill in the blanks.

Don’t ______ any dress material. Her dress is tight at the _______.

Options

A

waste; waste

waste; waste

B

waste; waist

waste; waist

C

waist; waist

waist; waist

D

waist; waste

waist; waste


Solution:

Correct Answer:

B

waste; waist

waste; waist


Question 203


Select the most appropriate option to fill in the blank.

I see a man outside, but I ______ that he will help us

Select the most appropriate option to fill in the blank.

I see a man outside, but I ______ that he will help us

Options

A

was doubting

was doubting

B

had doubted

had doubted

C

am doubting

am doubting

D

doubt

doubt


Solution:

Correct Answer:

D

doubt

doubt


Question 204


Select the most appropriate option to fill in the blank.

He constantly ______ to the thesaurus while writing his blogs.

Select the most appropriate option to fill in the blank.

He constantly ______ to the thesaurus while writing his blogs.

Options

A

referred

referred

B

reffered

reffered

C

refered

refered

D

refferred

refferred


Solution:

Correct Answer:

A

referred

referred


Question 205


Select the correct punctuation mark to fill in the blank.

He went his way ______ I, mine.

Select the correct punctuation mark to fill in the blank.

He went his way ______ I, mine.

Options

A

comma (,)

comma (,)

B

colon (:)

colon (:)

C

dash (-)

dash (-)

D

semicolon (;)

semicolon (;)


Solution:

Correct Answer:

D

semicolon (;)

semicolon (;)


Question 206


Select the most appropriate option to substitute the underlined word in the given sentence. If there is no need to substitute it, select ‘No substitution required’.

The government settled the mid-day meal scheme with great fanfare.

Select the most appropriate option to substitute the underlined word in the given sentence. If there is no need to substitute it, select ‘No substitution required’.

The government settled the mid-day meal scheme with great fanfare.

Options

A

stationed

stationed

B

installed

installed

C

No substitution required

No substitution required

D

launched

launched


Solution:

Correct Answer:

D

launched

launched


Question 207


Select the most appropriate ANTONYM of the given word.
Grave

Select the most appropriate ANTONYM of the given word.
Grave

Options

A

Significant

Significant

B

Substantial

Substantial

C

Slight

Slight

D

Serious

Serious


Solution:

Correct Answer:

C

Slight

Slight


Question 208


Select the most appropriate synonym of the given word.
Enigmatic

Select the most appropriate synonym of the given word.
Enigmatic

Options

A

Explicit

Explicit

B

Apathetic

Apathetic

C

Ambitious

Ambitious

D

Mysterious

Mysterious


Solution:

Correct Answer:

D

Mysterious

Mysterious


Question 209


Four sentences have been given below out of which three are alike in some manner and one is different. Select the one that is different.

Four sentences have been given below out of which three are alike in some manner and one is different. Select the one that is different.

Options

A

It took almost two hours to reach the top.

It took almost two hours to reach the top.

B

The mother Bear suddenly came charging with a half- growl, half scream of rage.

The mother Bear suddenly came charging with a half- growl, half scream of rage.

C

Barb saw immediately that it was a grizzly -the silvertip fur shining in the sunlight.

Barb saw immediately that it was a grizzly -the silvertip fur shining in the sunlight.

D

How can so huge an animal move so fast she thought.

How can so huge an animal move so fast she thought.


Solution:

Correct Answer:

A

It took almost two hours to reach the top.

It took almost two hours to reach the top.


Question 210


Select the most appropriate option to substitute the underlined word in the given sentence. If there is no need to substitute it, select ‘No substitution required’.

Her story was so aesthetic that we couldn’t help sympathising with her.

Select the most appropriate option to substitute the underlined word in the given sentence. If there is no need to substitute it, select ‘No substitution required’.

Her story was so aesthetic that we couldn’t help sympathising with her.

Options

A

No substitution required

No substitution required

B

pathetic

pathetic

C

synthetic

synthetic

D

sarcastic

sarcastic


Solution:

Correct Answer:

B

pathetic

pathetic


Question 211


Four sentences of a paragraph are given below in jumbled order. Arrange the sentences in the correct order to form a meaningful and coherent paragraph.

A. Razia Sultan was the first Muslim female ruler of the Delhi Sultanate.
B. It was historically important for her when she got the throne because the previous generations of her family, that is, her ancestors were not nobility.
C. She was the first woman who got the title of Sultan.
D. They were actually slaves.

Four sentences of a paragraph are given below in jumbled order. Arrange the sentences in the correct order to form a meaningful and coherent paragraph.

A. Razia Sultan was the first Muslim female ruler of the Delhi Sultanate.
B. It was historically important for her when she got the throne because the previous generations of her family, that is, her ancestors were not nobility.
C. She was the first woman who got the title of Sultan.
D. They were actually slaves.

Options

A

ABDC

ABDC

B

ADCB

ADCB

C

ABCD

ABCD

D

ACBD

ACBD


Solution:

Correct Answer:

D

ACBD

ACBD


Question 212


Select the option that expresses the given sentence in indirect speech.

Barb shouted, “Malcolm, hold on. I am going for help.”

Select the option that expresses the given sentence in indirect speech.

Barb shouted, “Malcolm, hold on. I am going for help.”

Options

A

Barb ordered Malcolm to hold on as he was going for help.

Barb ordered Malcolm to hold on as he was going for help.

B

Barb urgently told Malcolm to hold on as he was going for help.

Barb urgently told Malcolm to hold on as he was going for help.

C

Barb urgently told Malcolm to hold on as he is going for help.

Barb urgently told Malcolm to hold on as he is going for help.

D

Barb told Malcolm that hold on as I am going for help.

Barb told Malcolm that hold on as I am going for help.


Solution:

Correct Answer:

B

Barb urgently told Malcolm to hold on as he was going for help.

Barb urgently told Malcolm to hold on as he was going for help.


Question 213


Select the most appropriate option to collocate with the word ‘donkey’ to fill in the blank.

When the donkey started ______ everybody knew that it was no lion.

Select the most appropriate option to collocate with the word ‘donkey’ to fill in the blank.

When the donkey started ______ everybody knew that it was no lion.

Options

A

barking

barking

B

growling

growling

C

howling

howling

D

braying

braying


Solution:

Correct Answer:

D

braying

braying


Question 214


Select the most appropriate option to collocate with the word ‘goal’ to fill in the blank.

One really has to work hard to ______ one’s goal.

Select the most appropriate option to collocate with the word ‘goal’ to fill in the blank.

One really has to work hard to ______ one’s goal.

Options

A

obtain

obtain

B

acquire

acquire

C

achieve

achieve

D

take

take


Solution:

Correct Answer:

C

achieve

achieve


Question 215


Four sentences of a paragraph are given below in jumbled order. Arrange the sentences in the correct order to form a meaningful and coherent paragraph.

A. This was a young nomad, muffled in a long sheepskin coat and wearing a pigtail, as all Tibetan men who are not monks do.
B. He led us to his black tent made of yak's hair, where his wife was waiting for him.
C. The region through which we were traveling was completely unpopulated, and during the next eight days of our march, we met only one small caravan.
D. I have a vivid recollection of one person whom I encountered on this stretch of road.

Four sentences of a paragraph are given below in jumbled order. Arrange the sentences in the correct order to form a meaningful and coherent paragraph.

A. This was a young nomad, muffled in a long sheepskin coat and wearing a pigtail, as all Tibetan men who are not monks do.
B. He led us to his black tent made of yak's hair, where his wife was waiting for him.
C. The region through which we were traveling was completely unpopulated, and during the next eight days of our march, we met only one small caravan.
D. I have a vivid recollection of one person whom I encountered on this stretch of road.

Options

A

CBDA

CBDA

B

ADBC

ADBC

C

CDAB

CDAB

D

DACB

DACB


Solution:

Correct Answer:

C

CDAB

CDAB


Question 216


Four sentences of a paragraph are given below in jumbled order. Arrange the sentences in the correct order to form a meaningful and coherent paragraph.

A. One day there was a letter.
B. Probably this is a personal letter, so I did not read it.”
C. My secretary came up to me and said, “Madam, it seems to be from somebody who knows you well.
D. She placed the letter in front of me and left.

Four sentences of a paragraph are given below in jumbled order. Arrange the sentences in the correct order to form a meaningful and coherent paragraph.

A. One day there was a letter.
B. Probably this is a personal letter, so I did not read it.”
C. My secretary came up to me and said, “Madam, it seems to be from somebody who knows you well.
D. She placed the letter in front of me and left.

Options

A

DACB

DACB

B

ACBD

ACBD

C

CBDA

CBDA

D

ADBC

ADBC


Solution:

Correct Answer:

B

ACBD

ACBD


Question 217


Select the correctly punctuated sentence.

Select the correctly punctuated sentence.

Options

A

“Thanks,” said aditi and hurriedly went inside.

“Thanks,” said aditi and hurriedly went inside.

B

“Thanks,” said Aditi and hurriedly went inside.

“Thanks,” said Aditi and hurriedly went inside.

C

“Thanks.” said Aditi. And hurriedly went inside.

“Thanks.” said Aditi. And hurriedly went inside.

D

“Thanks, said Aditi,” and hurriedly went inside.

“Thanks, said Aditi,” and hurriedly went inside.


Solution:

Correct Answer:

B

“Thanks,” said Aditi and hurriedly went inside.

“Thanks,” said Aditi and hurriedly went inside.


Question 218


Select the most appropriate option to fill in the blank.

______ he knew all the questions he was slow in writing.

Select the most appropriate option to fill in the blank.

______ he knew all the questions he was slow in writing.

Options

A

Because

Because

B

In case

In case

C

In order that

In order that

D

Although

Although


Solution:

Correct Answer:

D

Although

Although


Question 219


Select the most appropriate option to fill in the blank.

I wonder whom ______ for guidance.

Select the most appropriate option to fill in the blank.

I wonder whom ______ for guidance.

Options

A

approach

approach

B

approached

approached

C

approaching

approaching

D

to approach

to approach


Solution:

Correct Answer:

D

to approach

to approach


Question 220


Select the most appropriate option to fill in the blank.

We ______ a survey of investor’s behaviour for our financial magazine.

Select the most appropriate option to fill in the blank.

We ______ a survey of investor’s behaviour for our financial magazine.

Options

A

carried on

carried on

B

carried away

carried away

C

carried off

carried off

D

carried out

carried out


Solution:

Correct Answer:

D

carried out

carried out


Question 221


Comprehension:
In the following passage, some words have been deleted. Read the passage carefully and select the most appropriate option to fill in each blank.

There are some historical mysteries that may never be (1)_______, from the date that Jesus was born to the identity (2)________ Jack the Ripper to the location of Cleopatra's tomb. (3)________, that's because the relevant excavated material has been (4)_______ or an archaeological site has been destroyed. Other times, it's (5)______ new evidence is unlikely to come forward or the surviving evidence is too vague to lead scholars to a consensus.
The lack of answers only makes these enigmas more intriguing.
SubQuestion No : 46
Select the most appropriate option to fill in blank No. 1.

Comprehension:
In the following passage, some words have been deleted. Read the passage carefully and select the most appropriate option to fill in each blank.

There are some historical mysteries that may never be (1)_______, from the date that Jesus was born to the identity (2)________ Jack the Ripper to the location of Cleopatra's tomb. (3)________, that's because the relevant excavated material has been (4)_______ or an archaeological site has been destroyed. Other times, it's (5)______ new evidence is unlikely to come forward or the surviving evidence is too vague to lead scholars to a consensus.
The lack of answers only makes these enigmas more intriguing.
SubQuestion No : 46
Select the most appropriate option to fill in blank No. 1.

Options

A

solving

solving

B

solved

solved

C

solves

solves

D

solve

solve


Solution:

Correct Answer:

B

solved

solved


Question 222


Comprehension:
In the following passage, some words have been deleted. Read the passage carefully and select the most appropriate option to fill in each blank.

There are some historical mysteries that may never be (1)_______, from the date that Jesus was born to the identity (2)________ Jack the Ripper to the location of Cleopatra's tomb. (3)________, that's because the relevant excavated material has been (4)_______ or an archaeological site has been destroyed. Other times, it's (5)______ new evidence is unlikely to come forward or the surviving evidence is too vague to lead scholars to a consensus.
The lack of answers only makes these enigmas more intriguing.
SubQuestion No : 47
Select the most appropriate option to fill in blank No. 2.

Comprehension:
In the following passage, some words have been deleted. Read the passage carefully and select the most appropriate option to fill in each blank.

There are some historical mysteries that may never be (1)_______, from the date that Jesus was born to the identity (2)________ Jack the Ripper to the location of Cleopatra's tomb. (3)________, that's because the relevant excavated material has been (4)_______ or an archaeological site has been destroyed. Other times, it's (5)______ new evidence is unlikely to come forward or the surviving evidence is too vague to lead scholars to a consensus.
The lack of answers only makes these enigmas more intriguing.
SubQuestion No : 47
Select the most appropriate option to fill in blank No. 2.

Options

A

of

of

B

at

at

C

to

to

D

for

for


Solution:

Correct Answer:

A

of

of


Question 223


Comprehension:
In the following passage, some words have been deleted. Read the passage carefully and select the most appropriate option to fill in each blank.

There are some historical mysteries that may never be (1)_______, from the date that Jesus was born to the identity (2)________ Jack the Ripper to the location of Cleopatra's tomb. (3)________, that's because the relevant excavated material has been (4)_______ or an archaeological site has been destroyed. Other times, it's (5)______ new evidence is unlikely to come forward or the surviving evidence is too vague to lead scholars to a consensus.
The lack of answers only makes these enigmas more intriguing.
SubQuestion No : 48
Select the most appropriate option to fill in blank No. 3.

Comprehension:
In the following passage, some words have been deleted. Read the passage carefully and select the most appropriate option to fill in each blank.

There are some historical mysteries that may never be (1)_______, from the date that Jesus was born to the identity (2)________ Jack the Ripper to the location of Cleopatra's tomb. (3)________, that's because the relevant excavated material has been (4)_______ or an archaeological site has been destroyed. Other times, it's (5)______ new evidence is unlikely to come forward or the surviving evidence is too vague to lead scholars to a consensus.
The lack of answers only makes these enigmas more intriguing.
SubQuestion No : 48
Select the most appropriate option to fill in blank No. 3.

Options

A

Here and there

Here and there

B

Sometimes

Sometimes

C

Always

Always

D

Consistently

Consistently


Solution:

Correct Answer:

B

Sometimes

Sometimes


Question 224


Comprehension:
In the following passage, some words have been deleted. Read the passage carefully and select the most appropriate option to fill in each blank.

There are some historical mysteries that may never be (1)_______, from the date that Jesus was born to the identity (2)________ Jack the Ripper to the location of Cleopatra's tomb. (3)________, that's because the relevant excavated material has been (4)_______ or an archaeological site has been destroyed. Other times, it's (5)______ new evidence is unlikely to come forward or the surviving evidence is too vague to lead scholars to a consensus.
The lack of answers only makes these enigmas more intriguing.
SubQuestion No : 49
Select the most appropriate option to fill in blank No. 4.

Comprehension:
In the following passage, some words have been deleted. Read the passage carefully and select the most appropriate option to fill in each blank.

There are some historical mysteries that may never be (1)_______, from the date that Jesus was born to the identity (2)________ Jack the Ripper to the location of Cleopatra's tomb. (3)________, that's because the relevant excavated material has been (4)_______ or an archaeological site has been destroyed. Other times, it's (5)______ new evidence is unlikely to come forward or the surviving evidence is too vague to lead scholars to a consensus.
The lack of answers only makes these enigmas more intriguing.
SubQuestion No : 49
Select the most appropriate option to fill in blank No. 4.

Options

A

present

present

B

found

found

C

distracted

distracted

D

lost

lost


Solution:

Correct Answer:

D

lost

lost


Question 225


Comprehension:
In the following passage, some words have been deleted. Read the passage carefully and select the most appropriate option to fill in each blank.

There are some historical mysteries that may never be (1)_______, from the date that Jesus was born to the identity (2)________ Jack the Ripper to the location of Cleopatra's tomb. (3)________, that's because the relevant excavated material has been (4)_______ or an archaeological site has been destroyed. Other times, it's (5)______ new evidence is unlikely to come forward or the surviving evidence is too vague to lead scholars to a consensus.
The lack of answers only makes these enigmas more intriguing.
SubQuestion No : 50
Select the most appropriate option to fill in blank No. 5.

Comprehension:
In the following passage, some words have been deleted. Read the passage carefully and select the most appropriate option to fill in each blank.

There are some historical mysteries that may never be (1)_______, from the date that Jesus was born to the identity (2)________ Jack the Ripper to the location of Cleopatra's tomb. (3)________, that's because the relevant excavated material has been (4)_______ or an archaeological site has been destroyed. Other times, it's (5)______ new evidence is unlikely to come forward or the surviving evidence is too vague to lead scholars to a consensus.
The lack of answers only makes these enigmas more intriguing.
SubQuestion No : 50
Select the most appropriate option to fill in blank No. 5.

Options

A

as

as

B

because

because

C

that

that

D

so

so


Solution:

Correct Answer:

B

because

because


Question 226


Comprehension:
Read the given passage and answer the questions that follow.

Indian onions are in huge demand, both in India and abroad thanks to their strong pungent flavour. In fact, more than 15 lakh metric tonnes of onions worth ₹2,826.50 crores were exported during the year 2020-21. Indian onions are usually made up of kharif and rabi onions. Kharif onions are planted between July to August and harvested between October to December. Rabi onions on the other hand are planted between December to January and harvested from March to April, the summer months. About 60% of onion production happens during rabi season and these onions are stored to meet domestic as well as export demand till the arrival of the kharif onion crop. As a general rule, rabi onions store well and are preferred for exports. This is because rabi onions tend to be dry and therefore can withstand transportation via rail, road or freight shipping.
Some of the most common varieties that are exported are White Onion, Red Onion, Bangalore Rose, Podisu and Krishnapuram Rose. Certain varieties of yellow onion such as Suprex, Granex 55, Granex 429, Arad- H, and Tana F1 are exported to European countries.
India exports both fresh and dehydrated onions to other countries. However, fresh onions constitute around 70% of the total onions exported from the country. Bangladesh is our biggest customer, consuming 30-35% of the total Indian onions exports. This is closely followed by Malaysia which imports Indian onions worth ₹400 to ₹500 crores every year. UAE comes in third with an annual import of onions worth ₹300 crores. The other major export destinations are Sri Lanka, Indonesia, Pakistan, Nepal and Singapore.
The huge demand for Indian onions is driven by the Indians settled in these countries. Bangladeshis have food preferences similar to Indians and therefore are huge consumers of Indian onions. Qatar and UAE have a huge percentage of Indians living there. A major chunk of these exported onions goes into the hotel, restaurant and catering sector of these countries.
SubQuestion No : 51
Which of the following is NOT true for rabi onions?

Comprehension:
Read the given passage and answer the questions that follow.

Indian onions are in huge demand, both in India and abroad thanks to their strong pungent flavour. In fact, more than 15 lakh metric tonnes of onions worth ₹2,826.50 crores were exported during the year 2020-21. Indian onions are usually made up of kharif and rabi onions. Kharif onions are planted between July to August and harvested between October to December. Rabi onions on the other hand are planted between December to January and harvested from March to April, the summer months. About 60% of onion production happens during rabi season and these onions are stored to meet domestic as well as export demand till the arrival of the kharif onion crop. As a general rule, rabi onions store well and are preferred for exports. This is because rabi onions tend to be dry and therefore can withstand transportation via rail, road or freight shipping.
Some of the most common varieties that are exported are White Onion, Red Onion, Bangalore Rose, Podisu and Krishnapuram Rose. Certain varieties of yellow onion such as Suprex, Granex 55, Granex 429, Arad- H, and Tana F1 are exported to European countries.
India exports both fresh and dehydrated onions to other countries. However, fresh onions constitute around 70% of the total onions exported from the country. Bangladesh is our biggest customer, consuming 30-35% of the total Indian onions exports. This is closely followed by Malaysia which imports Indian onions worth ₹400 to ₹500 crores every year. UAE comes in third with an annual import of onions worth ₹300 crores. The other major export destinations are Sri Lanka, Indonesia, Pakistan, Nepal and Singapore.
The huge demand for Indian onions is driven by the Indians settled in these countries. Bangladeshis have food preferences similar to Indians and therefore are huge consumers of Indian onions. Qatar and UAE have a huge percentage of Indians living there. A major chunk of these exported onions goes into the hotel, restaurant and catering sector of these countries.
SubQuestion No : 51
Which of the following is NOT true for rabi onions?

Options

A

Rabi onions are planted between July to August.

Rabi onions are planted between July to August.

B

Rabi onions can withstand transportation via rail, road or freight shipping.

Rabi onions can withstand transportation via rail, road or freight shipping.

C

Rabi onions are stored to meet domestic as well as export demand.

Rabi onions are stored to meet domestic as well as export demand.

D

Rabi onions store well and are preferred for exports.

Rabi onions store well and are preferred for exports.


Solution:

Correct Answer:

A

Rabi onions are planted between July to August.

Rabi onions are planted between July to August.


Question 227


Comprehension:
Read the given passage and answer the questions that follow.

Indian onions are in huge demand, both in India and abroad thanks to their strong pungent flavour. In fact, more than 15 lakh metric tonnes of onions worth ₹2,826.50 crores were exported during the year 2020-21. Indian onions are usually made up of kharif and rabi onions. Kharif onions are planted between July to August and harvested between October to December. Rabi onions on the other hand are planted between December to January and harvested from March to April, the summer months. About 60% of onion production happens during rabi season and these onions are stored to meet domestic as well as export demand till the arrival of the kharif onion crop. As a general rule, rabi onions store well and are preferred for exports. This is because rabi onions tend to be dry and therefore can withstand transportation via rail, road or freight shipping.
Some of the most common varieties that are exported are White Onion, Red Onion, Bangalore Rose, Podisu and Krishnapuram Rose. Certain varieties of yellow onion such as Suprex, Granex 55, Granex 429, Arad- H, and Tana F1 are exported to European countries.
India exports both fresh and dehydrated onions to other countries. However, fresh onions constitute around 70% of the total onions exported from the country. Bangladesh is our biggest customer, consuming 30-35% of the total Indian onions exports. This is closely followed by Malaysia which imports Indian onions worth ₹400 to ₹500 crores every year. UAE comes in third with an annual import of onions worth ₹300 crores. The other major export destinations are Sri Lanka, Indonesia, Pakistan, Nepal and Singapore.
The huge demand for Indian onions is driven by the Indians settled in these countries. Bangladeshis have food preferences similar to Indians and therefore are huge consumers of Indian onions. Qatar and UAE have a huge percentage of Indians living there. A major chunk of these exported onions goes into the hotel, restaurant and catering sector of these countries.
SubQuestion No : 52
The main theme of the above passage is:

Comprehension:
Read the given passage and answer the questions that follow.

Indian onions are in huge demand, both in India and abroad thanks to their strong pungent flavour. In fact, more than 15 lakh metric tonnes of onions worth ₹2,826.50 crores were exported during the year 2020-21. Indian onions are usually made up of kharif and rabi onions. Kharif onions are planted between July to August and harvested between October to December. Rabi onions on the other hand are planted between December to January and harvested from March to April, the summer months. About 60% of onion production happens during rabi season and these onions are stored to meet domestic as well as export demand till the arrival of the kharif onion crop. As a general rule, rabi onions store well and are preferred for exports. This is because rabi onions tend to be dry and therefore can withstand transportation via rail, road or freight shipping.
Some of the most common varieties that are exported are White Onion, Red Onion, Bangalore Rose, Podisu and Krishnapuram Rose. Certain varieties of yellow onion such as Suprex, Granex 55, Granex 429, Arad- H, and Tana F1 are exported to European countries.
India exports both fresh and dehydrated onions to other countries. However, fresh onions constitute around 70% of the total onions exported from the country. Bangladesh is our biggest customer, consuming 30-35% of the total Indian onions exports. This is closely followed by Malaysia which imports Indian onions worth ₹400 to ₹500 crores every year. UAE comes in third with an annual import of onions worth ₹300 crores. The other major export destinations are Sri Lanka, Indonesia, Pakistan, Nepal and Singapore.
The huge demand for Indian onions is driven by the Indians settled in these countries. Bangladeshis have food preferences similar to Indians and therefore are huge consumers of Indian onions. Qatar and UAE have a huge percentage of Indians living there. A major chunk of these exported onions goes into the hotel, restaurant and catering sector of these countries.
SubQuestion No : 52
The main theme of the above passage is:

Options

A

the onion market of India

the onion market of India

B

the export market of Indian onions

the export market of Indian onions

C

major importers of Indian onions

major importers of Indian onions

D

different crops of onions in India

different crops of onions in India


Solution:

Correct Answer:

B

the export market of Indian onions

the export market of Indian onions


Question 228


Comprehension:
Read the given passage and answer the questions that follow.

Indian onions are in huge demand, both in India and abroad thanks to their strong pungent flavour. In fact, more than 15 lakh metric tonnes of onions worth ₹2,826.50 crores were exported during the year 2020-21. Indian onions are usually made up of kharif and rabi onions. Kharif onions are planted between July to August and harvested between October to December. Rabi onions on the other hand are planted between December to January and harvested from March to April, the summer months. About 60% of onion production happens during rabi season and these onions are stored to meet domestic as well as export demand till the arrival of the kharif onion crop. As a general rule, rabi onions store well and are preferred for exports. This is because rabi onions tend to be dry and therefore can withstand transportation via rail, road or freight shipping.
Some of the most common varieties that are exported are White Onion, Red Onion, Bangalore Rose, Podisu and Krishnapuram Rose. Certain varieties of yellow onion such as Suprex, Granex 55, Granex 429, Arad- H, and Tana F1 are exported to European countries.
India exports both fresh and dehydrated onions to other countries. However, fresh onions constitute around 70% of the total onions exported from the country. Bangladesh is our biggest customer, consuming 30-35% of the total Indian onions exports. This is closely followed by Malaysia which imports Indian onions worth ₹400 to ₹500 crores every year. UAE comes in third with an annual import of onions worth ₹300 crores. The other major export destinations are Sri Lanka, Indonesia, Pakistan, Nepal and Singapore.
The huge demand for Indian onions is driven by the Indians settled in these countries. Bangladeshis have food preferences similar to Indians and therefore are huge consumers of Indian onions. Qatar and UAE have a huge percentage of Indians living there. A major chunk of these exported onions goes into the hotel, restaurant and catering sector of these countries.
SubQuestion No : 53
When are rabi onions harvested?

Comprehension:
Read the given passage and answer the questions that follow.

Indian onions are in huge demand, both in India and abroad thanks to their strong pungent flavour. In fact, more than 15 lakh metric tonnes of onions worth ₹2,826.50 crores were exported during the year 2020-21. Indian onions are usually made up of kharif and rabi onions. Kharif onions are planted between July to August and harvested between October to December. Rabi onions on the other hand are planted between December to January and harvested from March to April, the summer months. About 60% of onion production happens during rabi season and these onions are stored to meet domestic as well as export demand till the arrival of the kharif onion crop. As a general rule, rabi onions store well and are preferred for exports. This is because rabi onions tend to be dry and therefore can withstand transportation via rail, road or freight shipping.
Some of the most common varieties that are exported are White Onion, Red Onion, Bangalore Rose, Podisu and Krishnapuram Rose. Certain varieties of yellow onion such as Suprex, Granex 55, Granex 429, Arad- H, and Tana F1 are exported to European countries.
India exports both fresh and dehydrated onions to other countries. However, fresh onions constitute around 70% of the total onions exported from the country. Bangladesh is our biggest customer, consuming 30-35% of the total Indian onions exports. This is closely followed by Malaysia which imports Indian onions worth ₹400 to ₹500 crores every year. UAE comes in third with an annual import of onions worth ₹300 crores. The other major export destinations are Sri Lanka, Indonesia, Pakistan, Nepal and Singapore.
The huge demand for Indian onions is driven by the Indians settled in these countries. Bangladeshis have food preferences similar to Indians and therefore are huge consumers of Indian onions. Qatar and UAE have a huge percentage of Indians living there. A major chunk of these exported onions goes into the hotel, restaurant and catering sector of these countries.
SubQuestion No : 53
When are rabi onions harvested?

Options

A

December to January

December to January

B

March to April

March to April

C

October to December

October to December

D

July to August

July to August


Solution:

Correct Answer:

B

March to April

March to April


Question 229


Comprehension:
Read the given passage and answer the questions that follow.

Indian onions are in huge demand, both in India and abroad thanks to their strong pungent flavour. In fact, more than 15 lakh metric tonnes of onions worth ₹2,826.50 crores were exported during the year 2020-21. Indian onions are usually made up of kharif and rabi onions. Kharif onions are planted between July to August and harvested between October to December. Rabi onions on the other hand are planted between December to January and harvested from March to April, the summer months. About 60% of onion production happens during rabi season and these onions are stored to meet domestic as well as export demand till the arrival of the kharif onion crop. As a general rule, rabi onions store well and are preferred for exports. This is because rabi onions tend to be dry and therefore can withstand transportation via rail, road or freight shipping.
Some of the most common varieties that are exported are White Onion, Red Onion, Bangalore Rose, Podisu and Krishnapuram Rose. Certain varieties of yellow onion such as Suprex, Granex 55, Granex 429, Arad- H, and Tana F1 are exported to European countries.
India exports both fresh and dehydrated onions to other countries. However, fresh onions constitute around 70% of the total onions exported from the country. Bangladesh is our biggest customer, consuming 30-35% of the total Indian onions exports. This is closely followed by Malaysia which imports Indian onions worth ₹400 to ₹500 crores every year. UAE comes in third with an annual import of onions worth ₹300 crores. The other major export destinations are Sri Lanka, Indonesia, Pakistan, Nepal and Singapore.
The huge demand for Indian onions is driven by the Indians settled in these countries. Bangladeshis have food preferences similar to Indians and therefore are huge consumers of Indian onions. Qatar and UAE have a huge percentage of Indians living there. A major chunk of these exported onions goes into the hotel, restaurant and catering sector of these countries.
SubQuestion No : 54
Which country is the second biggest importer of Indian onions?

Comprehension:
Read the given passage and answer the questions that follow.

Indian onions are in huge demand, both in India and abroad thanks to their strong pungent flavour. In fact, more than 15 lakh metric tonnes of onions worth ₹2,826.50 crores were exported during the year 2020-21. Indian onions are usually made up of kharif and rabi onions. Kharif onions are planted between July to August and harvested between October to December. Rabi onions on the other hand are planted between December to January and harvested from March to April, the summer months. About 60% of onion production happens during rabi season and these onions are stored to meet domestic as well as export demand till the arrival of the kharif onion crop. As a general rule, rabi onions store well and are preferred for exports. This is because rabi onions tend to be dry and therefore can withstand transportation via rail, road or freight shipping.
Some of the most common varieties that are exported are White Onion, Red Onion, Bangalore Rose, Podisu and Krishnapuram Rose. Certain varieties of yellow onion such as Suprex, Granex 55, Granex 429, Arad- H, and Tana F1 are exported to European countries.
India exports both fresh and dehydrated onions to other countries. However, fresh onions constitute around 70% of the total onions exported from the country. Bangladesh is our biggest customer, consuming 30-35% of the total Indian onions exports. This is closely followed by Malaysia which imports Indian onions worth ₹400 to ₹500 crores every year. UAE comes in third with an annual import of onions worth ₹300 crores. The other major export destinations are Sri Lanka, Indonesia, Pakistan, Nepal and Singapore.
The huge demand for Indian onions is driven by the Indians settled in these countries. Bangladeshis have food preferences similar to Indians and therefore are huge consumers of Indian onions. Qatar and UAE have a huge percentage of Indians living there. A major chunk of these exported onions goes into the hotel, restaurant and catering sector of these countries.
SubQuestion No : 54
Which country is the second biggest importer of Indian onions?

Options

A

Bangladesh

Bangladesh

B

Malaysia

Malaysia

C

UAE

UAE

D

Sri Lanka

Sri Lanka


Solution:

Correct Answer:

B

Malaysia

Malaysia


Question 230


Comprehension:
Read the given passage and answer the questions that follow.

Indian onions are in huge demand, both in India and abroad thanks to their strong pungent flavour. In fact, more than 15 lakh metric tonnes of onions worth ₹2,826.50 crores were exported during the year 2020-21. Indian onions are usually made up of kharif and rabi onions. Kharif onions are planted between July to August and harvested between October to December. Rabi onions on the other hand are planted between December to January and harvested from March to April, the summer months. About 60% of onion production happens during rabi season and these onions are stored to meet domestic as well as export demand till the arrival of the kharif onion crop. As a general rule, rabi onions store well and are preferred for exports. This is because rabi onions tend to be dry and therefore can withstand transportation via rail, road or freight shipping.
Some of the most common varieties that are exported are White Onion, Red Onion, Bangalore Rose, Podisu and Krishnapuram Rose. Certain varieties of yellow onion such as Suprex, Granex 55, Granex 429, Arad- H, and Tana F1 are exported to European countries.
India exports both fresh and dehydrated onions to other countries. However, fresh onions constitute around 70% of the total onions exported from the country. Bangladesh is our biggest customer, consuming 30-35% of the total Indian onions exports. This is closely followed by Malaysia which imports Indian onions worth ₹400 to ₹500 crores every year. UAE comes in third with an annual import of onions worth ₹300 crores. The other major export destinations are Sri Lanka, Indonesia, Pakistan, Nepal and Singapore.
The huge demand for Indian onions is driven by the Indians settled in these countries. Bangladeshis have food preferences similar to Indians and therefore are huge consumers of Indian onions. Qatar and UAE have a huge percentage of Indians living there. A major chunk of these exported onions goes into the hotel, restaurant and catering sector of these countries.
SubQuestion No : 55
Why is there a huge demand of Indian onions in Middle East countries like UAE, Qatar etc?

Comprehension:
Read the given passage and answer the questions that follow.

Indian onions are in huge demand, both in India and abroad thanks to their strong pungent flavour. In fact, more than 15 lakh metric tonnes of onions worth ₹2,826.50 crores were exported during the year 2020-21. Indian onions are usually made up of kharif and rabi onions. Kharif onions are planted between July to August and harvested between October to December. Rabi onions on the other hand are planted between December to January and harvested from March to April, the summer months. About 60% of onion production happens during rabi season and these onions are stored to meet domestic as well as export demand till the arrival of the kharif onion crop. As a general rule, rabi onions store well and are preferred for exports. This is because rabi onions tend to be dry and therefore can withstand transportation via rail, road or freight shipping.
Some of the most common varieties that are exported are White Onion, Red Onion, Bangalore Rose, Podisu and Krishnapuram Rose. Certain varieties of yellow onion such as Suprex, Granex 55, Granex 429, Arad- H, and Tana F1 are exported to European countries.
India exports both fresh and dehydrated onions to other countries. However, fresh onions constitute around 70% of the total onions exported from the country. Bangladesh is our biggest customer, consuming 30-35% of the total Indian onions exports. This is closely followed by Malaysia which imports Indian onions worth ₹400 to ₹500 crores every year. UAE comes in third with an annual import of onions worth ₹300 crores. The other major export destinations are Sri Lanka, Indonesia, Pakistan, Nepal and Singapore.
The huge demand for Indian onions is driven by the Indians settled in these countries. Bangladeshis have food preferences similar to Indians and therefore are huge consumers of Indian onions. Qatar and UAE have a huge percentage of Indians living there. A major chunk of these exported onions goes into the hotel, restaurant and catering sector of these countries.
SubQuestion No : 55
Why is there a huge demand of Indian onions in Middle East countries like UAE, Qatar etc?

Options

A

The onions are used by hotels and restaurants.

The onions are used by hotels and restaurants.

B

Indian onions have a pungent taste.

Indian onions have a pungent taste.

C

These counties have a large Indian population.

These counties have a large Indian population.

D

Indian onions are cheap to import.

Indian onions are cheap to import.


Solution:

Correct Answer:

C

These counties have a large Indian population.

These counties have a large Indian population.


Types of Web Hosting

21-Apr-2023 06:03:43 | BLOG


types of web hosting


Read More

How to choose best web hosting

27-Sep-2022 09:46:18 | BLOG


best hosting


Read More

A2 Hosting Review

27-Sep-2022 09:45:14 | BLOG


a2 hosting


Read More

HostPapa Review

27-Sep-2022 09:44:24 | BLOG


hostpapa


Read More

Dreamhost Review

27-Sep-2022 09:43:44 | BLOG


dreamhost


Read More

Hostgator Review

27-Sep-2022 09:43:02 | BLOG


hostgator


Read More

Hostinger Review

27-Sep-2022 09:42:05 | BLOG


Hostinger


Read More

inMotion Hosting Review

27-Sep-2022 09:41:15 | BLOG


inmotion


Read More